Mandible Flashcards

1
Q

A 55-year-old woman is referred to the office by her dentist because of a 6-week history of exposed intraoral bone. She takes zoledronic acid for osteoporosis. Physical examination shows a 1-cm ulceration of gingiva with exposed necrotic bone adjacent to the right premolar. No infection or fistulization is noted. In addition to meticulous oral hygiene, which of the following is the most appropriate management?
A) Administration of prophylactic oral antibiotics
B) Curettage and bone grafting
C) Dental extraction
D) Segmental resection
E) Observation only

A

E) Observation only

The most appropriate management is observation only. The clinical vignette illustrates a case of Stage I bisphosphonate-related osteonecrosis of the jaw (BRONJ). These patients are typically asymptomatic, with the exception of exposed and/or necrotic bone. Antibiotics are not recommended unless there is infection (Stage II or III). Stage II BRONJ features exposed and/or necrotic bone with pain and local infection. Segmental resection is reserved for Stage III BRONJ. In general, the need for surgery is guided by the severity of the stage. Stage III BRONJ is characterized by exposed and/or necrotic bone with pain, infection, and the presence of another complication, such as osteolysis extending from the superior to the inferior border of the mandible, pathologic fracture, or extraoral fistula.

How well did you know this?
1
Not at all
2
3
4
5
Perfectly
2
Q

Stage I bisphosphonate-related osteonecrosis of the jaw

A

Stage I bisphosphonate-related osteonecrosis of the jaw (BRONJ). These patients are typically asymptomatic, with the exception of exposed and/or necrotic bone.

How well did you know this?
1
Not at all
2
3
4
5
Perfectly
3
Q

Stage I bisphosphonate-related osteonecrosis of the jaw

A

Observation

How well did you know this?
1
Not at all
2
3
4
5
Perfectly
4
Q

Stage II bisphosphonate-related osteonecrosis of the jaw: Management

A

Stage II BRONJ features exposed and/or necrotic bone with pain and local infection.

How well did you know this?
1
Not at all
2
3
4
5
Perfectly
5
Q

Stage II bisphosphonate-related osteonecrosis of the jaw

A

Antibiotics

How well did you know this?
1
Not at all
2
3
4
5
Perfectly
6
Q

Stage III bisphosphonate-related osteonecrosis of the jaw: Management

A

Stage III BRONJ is characterized by exposed and/or necrotic bone with pain, infection, and the presence of another complication, such as osteolysis extending from the superior to the inferior border of the mandible, pathologic fracture, or extraoral fistula.

How well did you know this?
1
Not at all
2
3
4
5
Perfectly
7
Q

Stage III bisphosphonate-related osteonecrosis of the jaw: Management

A

Segmental resection, antibiotics

How well did you know this?
1
Not at all
2
3
4
5
Perfectly
8
Q

A 33-year-old woman is brought to the emergency department after sustaining injuries in a motor vehicle collision. The patient notes pain on opening her mouth. Physical examination shows bilateral facial swelling and loss of posterior facial height. An anterior open bite is also noted. Which of the following additional findings on physical examination is most likely to suggest a bilateral subcondylar fracture in this patient?
A) Bilateral facial numbness
B) Bilateral mastoid ecchymosis
C) Blood in the external auditory canal
D) Clear fluid in the external auditory canal
E) Preauricular pain

A

E) Preauricular pain

Bilateral subcondylar fractures result in premature occlusion of the posterior teeth along with an anterior open bite, loss of posterior facial height, and bilateral facial swelling with pain on mouth opening. These findings occur because the subcondylar fracture interrupts the integrity of the vertical buttress. The lateral pterygoid muscles displace the condylar necks medially and anteriorly, allowing the unopposed vertical action of the temporalis and masseter muscles to shorten the posterior facial height. Swelling bilaterally would be expected in this fracture, and opening would displace the fracture line, causing pain.Bilateral facial numbness suggests a fracture of the ramus or body as the inferior alveolar nerve traverses these areas. Blood in the external auditory canal suggests a fracture more proximal than subcondylar. Bilateral mastoid ecchymosis and/or clear fluid in the external auditory canal suggest a skull base fracture.

How well did you know this?
1
Not at all
2
3
4
5
Perfectly
9
Q

Clinical presentation of bilateral subcondylar fracture

A

Bilateral subcondylar fractures result in premature occlusion of the posterior teeth along with an anterior open bite, loss of posterior facial height, and bilateral facial swelling with pain on mouth opening. Swelling bilaterally would be expected in this fracture, and opening would displace the fracture line, causing pain.

How well did you know this?
1
Not at all
2
3
4
5
Perfectly
10
Q

Anatomical reason behind clinical presentation of bilateral subcondylar fracture

A

These findings occur because the subcondylar fracture interrupts the integrity of the vertical buttress.

The lateral pterygoid muscles displace the condylar necks medially and anteriorly, allowing the unopposed vertical action of the temporalis and masseter muscles to shorten the posterior facial height.

How well did you know this?
1
Not at all
2
3
4
5
Perfectly
11
Q

Bilateral facial numbness suggests what injury?

A

Bilateral facial numbness suggests a fracture of the ramus or body as the inferior alveolar nerve traverses these areas.

How well did you know this?
1
Not at all
2
3
4
5
Perfectly
12
Q

Bilateral mastoid ecchymosis and/or clear fluid in the external auditory canal suggest what injury?

A

Bilateral mastoid ecchymosis and/or clear fluid in the external auditory canal suggest a skull base fracture.

How well did you know this?
1
Not at all
2
3
4
5
Perfectly
13
Q

Subcondylar fracture interrupts the integrity of the _______ buttress.

A

Subcondylar fracture interrupts the integrity of the vertical buttress.

How well did you know this?
1
Not at all
2
3
4
5
Perfectly
14
Q

A 25-year-old man is brought to the emergency department 2 hours after being punched in the left side of the face. Physical examination shows swelling on the left side of the face. The panoramic x-ray study (Panorex) shown was obtained (Fracture of the 3rd mandibular molar root). Which of the following is the most appropriate management?
A) Extraction of all components of the third left mandibular molar and maxillomandibular fixation (MMF) for 2 weeks
B) Extraction of all components of the third left mandibular molar and MMF for 6 weeks
C) Extraction of all components of the third left mandibular molar, MMF, and open reduction and internal fixation (ORIF)
D)Ligating the third left mandibular molar to the adjacent tooth for stability, MMF, and ORIF
E) Preservation of the third molar, MMF, and ORIF

A

C) Extraction of all components of the third left mandibular molar, MMF, and open reduction and internal fixation (ORIF)

The x-ray study shows a fracture of the root, thus the tooth and root must beremoved. Indications for extraction of a tooth in the line of a fracture include the need for MMF to regain the patient’s occlusion. The left parasymphyseal and comminuted left angle fractures can be managed in a number of ways, but the parasymphyseal fracture requires open reduction and internal fixation to prevent lateral displacement of the left mandibular body by the masseter muscle.

How well did you know this?
1
Not at all
2
3
4
5
Perfectly
15
Q

Management of parasymphyseal fractures

A

A parasymphyseal fracture requires open reduction and internal fixation to prevent lateral displacement of the left mandibular body by the masseter muscle.

How well did you know this?
1
Not at all
2
3
4
5
Perfectly
16
Q
A 2-month-old male infant is brought to the office because of mid face hypoplasia, craniosynostosis, and bilateral hand and foot anomalies. A photograph of the left foot is shown (syndactyly). This patient most likely has which of the following syndromes?
A) Apert
B) Crouzon
C) Goldenhar
D) Nager
E) Treacher Collins
A

A) Apert

The patient described has Apert syndrome. This autosomal dominant syndrome is characterized by bicoronal craniosynostosis that leads to turribrachycephaly, mid face hypoplasia, and complex hand and feet syndactyly.

How well did you know this?
1
Not at all
2
3
4
5
Perfectly
17
Q

Apert syndrome: Heredity

A

Autosomal dominant

How well did you know this?
1
Not at all
2
3
4
5
Perfectly
18
Q

Apert syndrome: Craniosynostosis

A

Bicoronal craniosynostosis that leads to turribrachycephaly

How well did you know this?
1
Not at all
2
3
4
5
Perfectly
19
Q

Apert syndrome: Associations

A

Bicoronal craniosynostosis that leads to turribrachycephaly, mid face hypoplasia, and complex hand and feet syndactyly.

How well did you know this?
1
Not at all
2
3
4
5
Perfectly
20
Q

Crouzon syndrome: Heredity

A

Autosomal dominant disorder

How well did you know this?
1
Not at all
2
3
4
5
Perfectly
21
Q

Crouzon syndrome: Craniosynostosis

A

Craniosynostosis involving the coronal, sagittal, and lambdoid sutures, as well as turribrachycephaly.

How well did you know this?
1
Not at all
2
3
4
5
Perfectly
22
Q

Crouzon syndrome: Associated findings

A

Craniosynostosis involving the coronal, sagittal, and lambdoid sutures, as well as turribrachycephaly. Other findings includemid face hypoplasia, exorbitism, and proptosis. The extremities are normal.

How well did you know this?
1
Not at all
2
3
4
5
Perfectly
23
Q

Goldenhar syndrome: Overview

A

Goldenhar syndrome, or oculoauriculovertebral dysplasia, involves asymmetry of the hard and soft tissues of the face.

How well did you know this?
1
Not at all
2
3
4
5
Perfectly
24
Q

Goldenhar syndrome: Laterality

A

This condition is most commonly unilateral but may be seen bilaterally in some patients.

How well did you know this?
1
Not at all
2
3
4
5
Perfectly
25
Q

Goldenhar syndrome: Manifestations

A

Manifestations of this syndrome include hypoplasia involving the mandible and underlying soft tissues of the face, epibulbar dermoids, and varied degrees of microtia on the affected side. Most patients have associated vertebral abnormalities.

How well did you know this?
1
Not at all
2
3
4
5
Perfectly
26
Q

Nager syndrome:

A

Characterized by craniofacial and upper extremity abnormalities:
Hypoplasia of the orbits, zygoma, maxilla, mandible, and soft palate. Auricular defects may also be present. Hypoplasia or agenesis occurs in the radius, thumbs, and metacarpals. Some patients may have radioulnar synostosis and elbow joint deformities.

How well did you know this?
1
Not at all
2
3
4
5
Perfectly
27
Q

Treacher Collins syndrome:

A

Patients with Treacher Collins syndrome, or mandibular dysostosis, have hypoplasia of the zygoma, maxilla, and mandible, downward slanting of the palpebral fissures, colobomas of the lower eyelids, absence of eyelashes, and auricular defects.

How well did you know this?
1
Not at all
2
3
4
5
Perfectly
28
Q

Nager syndrome: Heredity

A

Autosomal recessive

How well did you know this?
1
Not at all
2
3
4
5
Perfectly
29
Q

A 66-year-old man comes for a follow-up examination 7 months after resection of a T4 N1 M0 squamous cell carcinoma in the region of the retromolar trigone, including alveolectomy, followed by soft-tissue reconstruction with a platysma flap. Postoperatively, he received radiation therapy to the primary tumor site (6 Gy) and to the neck bilaterally (64 Gy). He completed radiation therapy 5 months ago. Examination today shows a malodorous, tender area of exposed, soft bone at the operative site. A panoramic x-ray study (Panorex) is shown. Multiple biopsies are negative for recurrent carcinoma. Which of the following is the most appropriate management?
A ) Long-term intravenous antibiotic therapy
B ) Open reduction and internal fixation
C ) Segmental resection and vascularized tissue transfer
D ) Sequestrectomy

A

C ) Segmental resection and vascularized tissue transfer

The patient described has osteoradionecrosis of the mandible, a complication that occurs in up to 40% of patients receiving adjuvant radiation therapy for head and neck malignancies. When more extensive destruction of the mandible is present, or when there is a pathologic fracture, as seen in the scenario described, resection of all the necrotic bone and soft tissue is indicated, followed by reconstruction with vascularized bone and soft tissue. Successful healing occurs in up to 80 to 90% of patients with more extensive disease when treated in this way.

How well did you know this?
1
Not at all
2
3
4
5
Perfectly
30
Q

Osteoradionecrosis of the jaw occurs in up to 40% of patients receiving adjuvant radiation therapy for head and neck malignancies

A

Osteoradionecrosis occurs in up to 40% of patients receiving adjuvant radiation therapy for head and neck malignancies

How well did you know this?
1
Not at all
2
3
4
5
Perfectly
31
Q

Osteoradionecrosis of the jaw occurs due to:

A

Hypoxia, hypovascularity, hypocellularity, and impaired collagen synthesis.

How well did you know this?
1
Not at all
2
3
4
5
Perfectly
32
Q

Definition of osteoradionecrosis

A

The traditional definition is an area of exposed, irradiated bone that is nonhealing over 3 months.

How well did you know this?
1
Not at all
2
3
4
5
Perfectly
33
Q

Osteoradionecrosis of the jaw: Management

A
  • Osteoradionecrosis limited to the alveolar ridge or mandible superior to the alveolar canal: Debridement and antibiotic therapy, plus or minus hyperbaric oxygen therapy, with soft-tissue reconstruction as needed
  • When more extensive destruction of the mandible is present, or when there is a pathologic fracture:
    Resection of all the necrotic bone and soft tissue is indicated, followed by reconstruction with vascularized bone and soft tissue.
How well did you know this?
1
Not at all
2
3
4
5
Perfectly
34
Q

Osteoradionecrosis of the jaw: Successful treatment in ____ patients with severe disease treated with resection and reconstruction with vascularized bone and soft tissue

A

Successful healing occurs in up to 80 to 90% of patients with more extensive disease when treated in this way.

How well did you know this?
1
Not at all
2
3
4
5
Perfectly
35
Q

Osteoradionecrosis of the jaw: Local flaps

A

Local flaps are of limited use for soft-tissue coverage because of the radiation. (RT to the neck)

How well did you know this?
1
Not at all
2
3
4
5
Perfectly
36
Q

A 40-year-old woman is referred for treatment after being found to have an idiopathic fracture on panoramic radiograph (Panorex), performed because of severe pain following a dental procedure. She has been treated for osteopenia related to multiple myeloma. A CT is shown. On evaluation, the patient has chronic pain and exposed intraoral bone. Which of the following is the most likely diagnosis?
A ) Bisphosphonate-related osteonecrosis
B ) Breast cancer metastasis
C ) Infectious osteomyelitis
D ) Myeloma-induced changes to the mandible
E ) Old unhealed fracture from trauma

A

A ) Bisphosphonate-related osteonecrosis

Bisphosphonates are routinely prescribed in the practice of medical oncology, and the incidence in these patients for BRON ranges between 1% and 10%. Becauseof the common use of these drugs in multiple myeloma, there have been many case reports of patients with BRON.

How well did you know this?
1
Not at all
2
3
4
5
Perfectly
37
Q

Bisphosphonates and relevance to malignancy

A

Bisphosphonates are clinically important for the treatment of hypercalcemia of malignancy and can reduce cancer-induced bone pain.

How well did you know this?
1
Not at all
2
3
4
5
Perfectly
38
Q

Frequency of bisphosphonate related osteonecrosis in patient receiving this medication for cancer

A

Ranges between 1%-10%.

How well did you know this?
1
Not at all
2
3
4
5
Perfectly
39
Q
A 32-year-old man comes to the emergency department because hehas had jaw pain and difficulty closing his mouth after he was punched in the face 30 minutes ago. A panoramic radiograph (Panorex- right mandibular angle fracture) of the lower face is shown. Which of the following clinical findings is consistent with this fracture pattern?
A ) Anteriorcrossbite
B ) Fracture of tooth No. 31
C ) Maxillary occlusal cant
D ) Paresthesia of right upper lip
E ) Posterior open bite on the left
A

E ) Posterior open bite on the left

A right mandibular angle fracture is shown. Foreshortening of the height on the right from the fracture and powerful influence of the masseter will likely result in posterior open biteon the contralateral side. Anterior crossbite occurs in maxillary hypoplasia and prognathism conditions and is not a result of the fracture in the patient described. Standard numbering of the dentition of the maxillary dental arch from right to left is No. 1 to No. 16. The mandibular dental arch from left to right is No. 17 through No. 32. Maxillary occlusal cant occurs congenitally or with maxillary fracture.Paresthesias of the upper lip are not associated with mandibular fracture through the inferior alveolar nerve. This fracture would be consistent with paresthesia of the right lower lip.

How well did you know this?
1
Not at all
2
3
4
5
Perfectly
40
Q

Standard numbering of the dentition

A

Maxillary dental arch from right to left is No. 1 to No. 16.

Mandibular dental arch from left to right is No. 17 through No. 32.

How well did you know this?
1
Not at all
2
3
4
5
Perfectly
41
Q

A 56-year-old woman with type 1 diabetes mellitus has a six-month history of a slowly enlarging, painless mass in the mandible. She is edentulous. Physical examination shows a 4-cm mass on both the buccal and lingual portions of the alveolus with intact mucosa. A panoramic radiograph (Panorex) is shown. Biopsy confirms ameloblastoma. Which of the following is the most definitive management?
A ) Conservative segmental resection of the mandible, followed by nonvascularized iliac crest cortical bone grafting
B ) Curettage and cancellous bone grafting
C ) Segmental resection of the mandible with wide margins and cervical lymph node dissection, followed by vascularized free fibular bone grafting
D ) Segmental resection of the mandible with wide margins and cervical lymph nodedissection, followed by vascularized free fibular bone grafting and adjuvant radiation therapy
E ) Segmental resection of the mandible with wide margins, followed by vascularized free fibular bone grafting

A

E ) Segmental resection of the mandible with wide margins, followed by vascularized free fibular bone grafting

The history and radiographic findings in the patient described are most consistent with multicystic ameloblastoma. This odontogenic tumor commonly presents as a painless enlarging mass in the mandible or, less commonly, in the maxilla. The typical radiographic finding is a multilocular lucency with preservation of the cortex, though the cortex may be thinned.

Extraosseous lesions may have no radiographic findings. This tumor goes beyond the boundaries seen on radiograph.

Wide resection is necessary for multicystic lesions to prevent recurrence. The sizeable defect resulting from wide resection in the patient described would be best treated with vascularized bone graft, especially in light of her diabetes.

The lack of mucosal ulceration or cortical bone erosion in a lesion this size point away from a malignancy; therefore, cervical lymph node dissection and adjuvant radiation therapy would not be indicated.

How well did you know this?
1
Not at all
2
3
4
5
Perfectly
42
Q

Multicystic ameloblastoma: clinical presentation

A

This odontogenic tumor commonly presents as a painless enlarging mass in the mandible or, less commonly, in the maxilla.

How well did you know this?
1
Not at all
2
3
4
5
Perfectly
43
Q

Multicystic ameloblastoma: Radiographic findings

A

The typical radiographic finding is a multilocular lucency with preservation of the cortex, though the cortex may be thinned.

Extraosseous lesions may have no radiographic findings. This tumor goes beyond the boundaries seen on radiograph.

How well did you know this?
1
Not at all
2
3
4
5
Perfectly
44
Q

Multicystic ameloblastoma: Management

A

Wide resection is necessary for multicystic lesions to prevent recurrence.

Curettage or conservative resection of a multi cystic tumor would result in recurrence in 50% of patients. Conservative resection may have higher cure rates for unicystic lesions.

How well did you know this?
1
Not at all
2
3
4
5
Perfectly
45
Q

A 5-year-old girl is brought to the emergency department immediately after falling from a swing set and hitting her chin on the cement pavement. Physical examination shows a laceration of the chin and an anterior open bite. Radiographs show bilateral subcondylar fractures with medial displacement of the right condylar neck and comminution of the left condylar head. A good dental relation is established while using anesthesia. Which of the following is the most appropriate management?
A ) Intermaxillary fixation for one to two weeks
B ) Intermaxillary fixation for four to six weeks
C ) Open reduction and internal fixation of the right and left fractures followed by early range of motion exercises
D ) Open reduction and internal fixation of the right subcondylar fracture followed by intermaxillary fixation for one to two weeks
E ) Open reduction and internal fixation of the right subcondylar fracture followed by intermaxillary fixation for four to six weeks

A

A ) Intermaxillary fixation for one to two weeks

Because the condylar head is not displaced in the patient described, open reduction and internal fixation of the right subcondylar fracture is not indicated. Intermaxillary fixation should be applied and remain in place for one to two weeks after the surgery. Subsequently, active and passive physical therapy of the mandible should be performed to work the mandible and remold the subcondylar union. This is especially important because of the comminuted intracapsular fracture on the contralateral side. Intermaxillary fixation applied for an extended period (greater than four weeks) may result in postoperative ankylosis. This is very difficult to treat. Releasing the intermaxillary fixation at two weeks may cause a delayed crossbite or anterior open bite. Both may be treated secondarily with a sagittal split osteotomy

How well did you know this?
1
Not at all
2
3
4
5
Perfectly
46
Q

Typically, open reduction of a fracture of the mandibular condyle:

A

Open reduction of a fracture of the mandibular condyle is not commonly performed because the procedure may be complicated, and closed reduction is usually sufficient.

How well did you know this?
1
Not at all
2
3
4
5
Perfectly
47
Q

Open reduction of the condyle is indicated in what situations?

A

Open reduction of the condyle is indicated in the following four situations:
•Displacement into the middle cranial fossa
•Impossibility of obtaining adequate dental occlusion by closed reduction
•Lateral extracapsular displacement of the condyle
•Invasion by a foreign body (eg, a bullet from a gunshot wound)

How well did you know this?
1
Not at all
2
3
4
5
Perfectly
48
Q
A patient with bilateral displaced subcondylar fractures is most likely to have which of the following occlusal patterns?
A ) Anterior crossbite
B ) Anterior open bite
C ) Posterior crossbite
D ) Posterior open bite
A

B ) Anterior open bite

When a patient sustains a bilateral subcondylar fracture, there is the possibility of shortening of the posterior mandibular height. This shortening is secondary to telescoping of the condylar fragments and muscular pull of the boney components.

A premature occlusal contact of the posterior occlusion occurs, resulting in an anterior open bite.

How well did you know this?
1
Not at all
2
3
4
5
Perfectly
49
Q
A 6-year-old boy is evaluated because of severe pain on opening his mouth. One week ago, he hit his chin in a fall. Physical examination shows chin deviation to the right and premature contact of the molar region on the right. Facial radiographs are ordered. The most likely cause of these findings is fracture of which of the following segments of the mandible?
(A)Angle
(B)Body
(C)Condyle
(D)Ramus
(E)Symphysis
A

(C)Condyle

Mandibular condyle fractures are the most common facial fractures seen in children. In a recent review of more than 1250 pediatric maxillofacial fractures, condylar injuries constituted 34.9% of all mandible fractures; however, the angle (17.7%), body (14.9%), ramus (8%), and symphysis (9%) are also important sites of potential mandible fracture.

How well did you know this?
1
Not at all
2
3
4
5
Perfectly
50
Q

Most common pediatric facial fracture:

A

Mandibular

How well did you know this?
1
Not at all
2
3
4
5
Perfectly
51
Q

Most common sites of mandibular fracture in children

A

35%: Condylar
18%: Angle
15%: Body
8%: Ramus

How well did you know this?
1
Not at all
2
3
4
5
Perfectly
52
Q
Which of the following regions of the adult mandible has the highest incidence of fracture?
(A)Angle
(B)Body
(C)Coronoid
(D)Ramus
(E)Symphysis
A

(A)Angle

The angle of the mandible has the highest incidence of fracture—up to 30%. Incidence of fracture is 15% to 20% for the body and parasymphysis. The presence of an isolated symphyseal or parasymphyseal fracture should alert the surgeon to the possibility of a second fracture near the angle.

How well did you know this?
1
Not at all
2
3
4
5
Perfectly
53
Q

Most common sites of mandibular fracture in adults

A

30%: Angle
20%: Parasymphysis
15%: Body

How well did you know this?
1
Not at all
2
3
4
5
Perfectly
54
Q

The presence of an isolated symphyseal or parasymphyseal fracture should alert the surgeon to the possibility of:

A

The presence of an isolated symphyseal or parasymphyseal fracture should alert the surgeon to the possibility of a second fracture near the angle.

How well did you know this?
1
Not at all
2
3
4
5
Perfectly
55
Q
Which of the following muscles controls depression and protrusion of the mandible?
(A)Buccinator
(B)Digastric
(C)Lateral pterygoid
(D)Masseter
(E)Temporalis
A

(C)Lateral pterygoid

The lateral pterygoid muscle originates from the greater wing of the sphenoid, the inferotemporal crest, and the lateral pterygoid plate and inserts on the neck of the mandibular condyle and articular disc of the temporomandibular joint.

Its action is to depress, protrude, and move the mandible from side to side.

How well did you know this?
1
Not at all
2
3
4
5
Perfectly
56
Q

The muscles of mastication are all inserted upon the ________ and innervated by:

A

The muscles of mastication are all inserted upon the mandible and innervated by the mandibular division of the trigeminal (V) nerve

How well did you know this?
1
Not at all
2
3
4
5
Perfectly
57
Q

Anatomy of the lateral pterygoid muscle

A

The lateral pterygoid muscle originates from the greater wing of the sphenoid, the inferotemporal crest, and the lateral pterygoid plate and inserts on the neck of the mandibular condyle and articular disc of the temporomandibular joint.

How well did you know this?
1
Not at all
2
3
4
5
Perfectly
58
Q

Action of the lateral pterygoid muscle

A

Its action is to depress, protrude, and move the mandible from side to side.

How well did you know this?
1
Not at all
2
3
4
5
Perfectly
59
Q

Anatomy of the masseter

A

The masseter muscle originates from the zygomatic arch and inserts on the mandibular angle, ramus, and condyle.

How well did you know this?
1
Not at all
2
3
4
5
Perfectly
60
Q

Action of the masseter

A

Its action is to close the jaw.

How well did you know this?
1
Not at all
2
3
4
5
Perfectly
61
Q

Anatomy of the medial pterygoid muscle

A

The medial pterygoid muscle originates on the medial aspect of the lateral pterygoid plate of the sphenoid in the tuberosity of the maxilla. It inserts on the medial surface of the ramus of the mandible.

How well did you know this?
1
Not at all
2
3
4
5
Perfectly
62
Q

Action of the medial pterygoid muscle

A

Acts to close the jaw.

How well did you know this?
1
Not at all
2
3
4
5
Perfectly
63
Q

Anatomy of the temporalis muscle

A

The temporalis muscle originates from the temporal fascia and entire temporal fossa. It inserts on the coronoid process and the anterior border of the ramus of the mandible

How well did you know this?
1
Not at all
2
3
4
5
Perfectly
64
Q

Action of the temporalis muscle

A

Acts to close and retract the jaw.

How well did you know this?
1
Not at all
2
3
4
5
Perfectly
65
Q

Innervation of the buccinator muscle

A

The buccinator muscle is innervated by the facial nerve (VII).

How well did you know this?
1
Not at all
2
3
4
5
Perfectly
66
Q
A 20-year-old man comes to the office for consultation regarding malocclusion. On physical examination, the mandibular incisors are anterior to the maxillary incisors. The mesial buccal cusp of the maxillary first molar lies distal to the buccal groove of the mandibular first molar. These findings are characteristic of which of the following Angle classifications?
(A)Angle class I
(B)Angle class II, division I
(C)Angle class II, division II
(D)Angle class III
A

(D)Angle class III

Angle class I, or normal occlusion, is characterized as having the mesial buccal cusp of the upper first molar occluding in the buccal groove of the mandibular first molar. In Angle class II, both divisions I and II, the mandibular dentition is distal to its class I position. Class II, division 1 is lingually inclined, and Class II, division 2 is labially inclined. In Angle class III malocclusion, the mandibular molar is anterior to its normal position with the maxillary molar.

How well did you know this?
1
Not at all
2
3
4
5
Perfectly
67
Q

Angle class I

A

Angle class I, or normal occlusion, is characterized as having the mesial buccal cusp of the upper first molar occluding in the buccal groove of the mandibular first molar.

How well did you know this?
1
Not at all
2
3
4
5
Perfectly
68
Q

Angle class II

A

In Angle class II, both divisions I and II, the mandibular dentition is distal to its class I position.

How well did you know this?
1
Not at all
2
3
4
5
Perfectly
69
Q

Angle class II, division 1

A

Class II, division 1 is lingually inclined, and

How well did you know this?
1
Not at all
2
3
4
5
Perfectly
70
Q

Angle class II, division 2

A

Class II, division 2 is labially inclined.

How well did you know this?
1
Not at all
2
3
4
5
Perfectly
71
Q

Angle class III

A

In Angle class III malocclusion, the mandibular molar is anterior to its normal position with the maxillary molar.

How well did you know this?
1
Not at all
2
3
4
5
Perfectly
72
Q
A 46-year-old man comes to the office because he has pain in the jaw and trismus after being involved in a motor vehicle collision two days ago. Radiographs show a mandibular fracture. In adults, the normal range of vertical mandibular opening is closest to which of the following?
(A)11 to 20 mm
(B)21 to 30 mm
(C)31 to 40 mm
(D)41 to 50 mm
(E)51 to 60 mm
A

(D)41 to 50 mm

In adults, the vertical mandibular opening measured from maxillary incisal edge to mandibular incisal edge (interincisal distance) typically ranges from 40 to 50 mm. In addition, normal range of motion of the mandible includes lateral jaw excursion (measured at the midline incisor) to 10 mm on each side.
Decreased mandibular opening may indicate dysfunction of the temporomandibular joint (TMJ) or surrounding soft tissues. Patients who may potentially have internal derangement of the TMJ also may experience painless clicking when opening the mouth.

How well did you know this?
1
Not at all
2
3
4
5
Perfectly
73
Q

In adults, the vertical mandibular opening measured from maxillary incisal edge to mandibular incisal edge (interincisal distance) typically ranges from:

A

In adults, the vertical mandibular opening measured from maxillary incisal edge to mandibular incisal edge (interincisal distance) typically ranges from 40 to 50 mm.

How well did you know this?
1
Not at all
2
3
4
5
Perfectly
74
Q

In addition, normal range of motion of the mandible includes lateral jaw excursion to:

A

In addition, normal range of motion of the mandible includes lateral jaw excursion (measured at the midline incisor) to 10 mm on each side.

How well did you know this?
1
Not at all
2
3
4
5
Perfectly
75
Q

Dysfunction of the temporomandibular joint

A

Decreased mandibular opening may indicate dysfunction of the temporomandibular joint (TMJ) or surrounding soft tissues. Patients who may potentially have internal derangement of the TMJ also may experience painless clicking when opening the mouth.

How well did you know this?
1
Not at all
2
3
4
5
Perfectly
76
Q

For normal range of motion of the mandible, lateral jaw excursion is measured at:

A

Measured at the midline incisor

How well did you know this?
1
Not at all
2
3
4
5
Perfectly
77
Q
In planning open reduction and internal fixation in a patient with a low subcondylar neck fracture, which of the following extraoral incisions provides the safest and most versatile exposure to thefracture site?
(A) Postauricular
(B) Preauricular
(C) Retromandibular
(D) Submandibular
A

(C) Retromandibular

The retromandibular incision provides the safest and most versatile exposure for open reduction and internal fixation of submandibular fractures. When compared with the subcondylar and preauricular incisions, there is significantly less injury to the marginal mandibular, temporal, and zygomatic branches of the facial nerve.

The retromandibular incision allows access superiorly to the coronoid notch and inferiorly to the angle of the mandible. The addition of a transfacial trocar to this approach facilitates access to higher level subcondylar fractures as well. The preauricular, postauricular, and submandibular incisions provide a more limited view of low subcondylar fractures.

How well did you know this?
1
Not at all
2
3
4
5
Perfectly
78
Q

Which incision provides the safest and most versatile exposure for open reduction and internal fixation of submandibular fractures?

A

The retromandibular incision provides the safest and most versatile exposure for open reduction and internal fixation of submandibular fractures.

How well did you know this?
1
Not at all
2
3
4
5
Perfectly
79
Q

Retromandibular incision versus subcondylar and preauricular incision: morbidity

A

When compared with the subcondylar and preauricular incisions, with a retromandibular incision there is significantly less injury to the marginal mandibular, temporal, and zygomatic branches of the facial nerve.

How well did you know this?
1
Not at all
2
3
4
5
Perfectly
80
Q

The retromandibular incision allows access to:

A

The retromandibular incision allows access superiorly to the coronoid notch and inferiorly to the angle of the mandible.

How well did you know this?
1
Not at all
2
3
4
5
Perfectly
81
Q

With the addition of a transfacial tracer, the retromandibular incision allows access to:

A

The retromandibular incision allows access superiorly to the coronoid notch and inferiorly to the angle of the mandible. The addition of a transfacial trocar to this approach facilitates access to higher level subcondylar fractures as well.

How well did you know this?
1
Not at all
2
3
4
5
Perfectly
82
Q
A 5-year-old child has malocclusion and limited opening of the mouth after falling from playground equipment. A CT scan is shown. Which of the following is the most likely long-term sequela of this patient’s injury?
(A) Bimaxillary prognathism
(B) Mandibular hypoplasia
(C) Mandibular prognathism
(D) Maxillary hypoplasia
(E) Maxillary prognathism
A

(B) Mandibular hypoplasia

This child is at increased risk for mandibular hypoplasia. The condyle serves as a growth center for the mandible and contributes primarily to vertical growth. The condylar cartilage is a site of secondary passive growth dependent on forces acting on it, notably the medial and lateral pterygoid muscles. Pediatric condylar fractures generally remodel and do not often cause growth disturbance. However, the thin, localized functional matrix of the condyle may disallow normal mandibular growth after it has been injured and may result in unilateral or bilateral hypoplasia depending on the injury. Pediatric mandibular fractures are frequently treated conservatively via closed reduction and short periods of maxillomandibular fixation.

How well did you know this?
1
Not at all
2
3
4
5
Perfectly
83
Q

Growth center for the mandible

A

The condyle serves as a growth center for the mandible and contributes primarily to vertical growth. The condylar cartilage is a site of secondary passive growth dependent on forces acting on it, notably the medial and lateral pterygoid muscles.

How well did you know this?
1
Not at all
2
3
4
5
Perfectly
84
Q

Growth after a pediatric condylar fracture

A

Pediatric condylar fractures generally remodel and do not often cause growth disturbance. However, the thin, localized functional matrix of the condyle may disallow normal mandibular growth after it has been injured and may result in unilateral or bilateral hypoplasia depending on the injury.

How well did you know this?
1
Not at all
2
3
4
5
Perfectly
85
Q

A 12-year-old boy is brought to the emergency department after he fell while riding his bike and landed on the chin. Panorex radiographs show a minimally displaced fracture of the high right condylar neck and an open left parasymphyseal fracture. Which of the following is the most effective management?
(A) Observation with serial radiographs and restriction to soft diet
(B) Intermaxillary fixation with infraorbital and circummandibular wires for four weeks
(C) Intermaxillary fixation with arch bars for four weeks
(D) Open reduction with internal fixation of the parasymphyseal fracture and arch bars for two weeks
(E) Open reduction with internal fixation of both the parasymphyseal fracture and the fracture of the high condylar neck

A

(D) Open reduction with internal fixation of the parasymphyseal fracture and arch bars for two weeks

Observation alone is inadequate management of the parasymphyseal fracture. This child presents in the late phase of the mixed dentition and should have enough adult dentition in place to secure the arch bars. Wires to reduce the fractures will not provide as much stability as arch bars. This technique, however, is useful in the child who does not yet have enough adult dentition to secure the arch bars.
Immobilization for a short period (i.e., two weeks) is the appropriate management of the condylar neck fracture. This will help to allow the fractures to become stable enough to maintain the reduction once movement is instituted. Early movement helps to decrease the risk of ankylosis of the temporal mandibular joint.

Open reduction and internal fixation (ORIF) of the parasymphyseal fracture will ensure an anatomic reduction of a stable skeletal unit. This, in turn, will decrease the risk of infection and nonunion

How well did you know this?
1
Not at all
2
3
4
5
Perfectly
86
Q

Using wires instead of arch bars in a pediatric patient

A

Wires to reduce the fractures will not provide as much stability as arch bars. This technique, however, is useful in the child who does not yet have enough adult dentition to secure the arch bars.

How well did you know this?
1
Not at all
2
3
4
5
Perfectly
87
Q

Management of a condylar neck fracture

A

Immobilization for a short period (i.e., two weeks) is the appropriate management of the condylar neck fracture. This will help to allow the fractures to become stable enough to maintain the reduction once movement is instituted. Early movement helps to decrease the risk of ankylosis of the temporal mandibular joint.

How well did you know this?
1
Not at all
2
3
4
5
Perfectly
88
Q
A 22-year-old man sustains a left subcondylar fracture of the mandible during a motor vehicle collision. On CT scan, the condyle is displaced medially and anteriorly. This displacement is most likely caused by tension from which of the following muscles?
(A) Medial pterygoid
(B) Lateral pterygoid
(C) Masseter
(D) Mylohyoid
(E) Temporalis
A

(B) Lateral pterygoid

The inferior belly of the lateral pterygoid originates from the lateral pterygoid plate and inserts onto the scaphoid fossa of the condyle and joint capsule. The superior belly of the lateral pterygoid muscle originates from the sphenoid and inserts on the temporomandibular joint. The effect of the lateral pterygoid muscle is to displace the condyle medially and anteriorly in fractures of the condylar neck. It also tends to displace the meniscus anteriorly.

How well did you know this?
1
Not at all
2
3
4
5
Perfectly
89
Q

The mylohyoid muscle displaces segmental body fractures in what direction?

A

The mylohyoid muscle inserts on the body of the mandible, displacing segmental body fractures medially.

How well did you know this?
1
Not at all
2
3
4
5
Perfectly
90
Q

The effect of the lateral pterygoid muscle is to displace fractures of the condylar neck in what direction?

A

The effect of the lateral pterygoid muscle is to displace the condyle medially and anteriorly in fractures of the condylar neck. It also tends to displace the meniscus anteriorly.

How well did you know this?
1
Not at all
2
3
4
5
Perfectly
91
Q

A 22-year-old man sustains a transverse, noncomminuted fracture of the right mandibular angle when he is struck in the face during a fistfight. Which of the following interventions best adheres to Champy’s principle for management of this fracture?
(A) Dynamic compression plate with bicortical screws on the inferior edge of the mandible and a superior tension band
(B) Dynamic compression plate with bicortical screws and a mandibular arch bar
(C) Lag screw
(D) Miniplate with monocortical screws along the external oblique ridge
(E) Reconstruction plate with bicortical screws

A

(D) Miniplate with monocortical screws along the external oblique ridge

Champy’s principles for fracture management call for placement of miniplates along the lines of tension in the mandible at the site of the fracture. Because compression is not necessary, the miniplates can be anchored with monocortical screws. Based on the muscular forces pulling on the mandible, Champy determined that, anterior to the canine tooth, two miniplates are needed to control the rotational forces of the genial and digastric muscles;posterior to the canine tooth, just one miniplate is required.

In a 10-year review examining various methods for treating fractures of the mandibular angle, Ellis concluded that use of a single 2.0-mm noncompression miniplate was associated with fewer complications than a double-plate system (one using two compression or noncompression plates) or a reconstruction plate.

How well did you know this?
1
Not at all
2
3
4
5
Perfectly
92
Q

Champy’s principles for fracture management call for placement of miniplates where?

A

Champy’s principles for fracture management call for placement of miniplates along the lines of tension in the mandible at the site of the fracture.

How well did you know this?
1
Not at all
2
3
4
5
Perfectly
93
Q

Champy: When are one versus two miniplates required?

A

Based on the muscular forces pulling on the mandible, Champy determined that, anterior to the canine tooth, two miniplates are needed to control the rotational forces of the genial and digastric muscles; posterior to the canine tooth, just one miniplate is required.

How well did you know this?
1
Not at all
2
3
4
5
Perfectly
94
Q

Fractures of the mandibular angle: One versus two plates

A

In a 10-year review examining various methods for treating fractures of the mandibular angle, Ellis concluded that use of a single 2.0-mm noncompression miniplate was associated with fewer complications than a double-plate system (one using two compression or noncompression plates) or a reconstruction plate.

How well did you know this?
1
Not at all
2
3
4
5
Perfectly
95
Q

A 26-year-old man comes to the office because he has pain in the mandible seven days after undergoing open reduction and internal fixation of a fracture of the mandible. Physical examination shows infection in the submandibular space. Which of the following teeth are the most likely source of this infection?
(A) Mandibular canines
(B) Mandibular central incisors
(C) Mandibular first and second premolars
(D) Mandibular second and third molars
(E) Maxillary second and third molars

A

(D) Mandibular second and third molars

The submandibular space is located inferolateral to the mylohyoid muscle and superior to the hyoid bone. The contents of the submandibular space include the submandibular gland, lymph nodes, the facial vein and artery, and the inferior loop of the hypoglossal (XII) nerve. Anteriorly, the submandibular space communicates with the submental space and posteriorly with the pharyngeal space. The sublingual space is located superomedial to the mylohyoid muscle. Involvement of the submandibular space is produced principally by infections of the second and third mandibular molars because of the more superior position of the mylohyoid ridge on the mandible posteriorly, which places the root apices of the second and third molars beneath the mylohyoid muscle. Infections of the maxillary molars, when they extend through the buccal cortical plates above the attachments of the buccinator muscle, can present as infections of the buccal space. Infections from the anterior mandibular teeth (anterior to the second molar) usually drain above the mylohyoid muscle into the sublingual space.

How well did you know this?
1
Not at all
2
3
4
5
Perfectly
96
Q

The submandibular space

A

The submandibular space is located inferolateral to the mylohyoid muscle and superior to the hyoid bone.

How well did you know this?
1
Not at all
2
3
4
5
Perfectly
97
Q

Contents of the submandibular space

A

The contents of the submandibular space include the submandibular gland, lymph nodes, the facial vein and artery, and the inferior loop of the hypoglossal (XII) nerve.

How well did you know this?
1
Not at all
2
3
4
5
Perfectly
98
Q

The submandibular space communicates with?

A

Anteriorly, the submandibular space communicates with the submental space and posteriorly with the pharyngeal space.

How well did you know this?
1
Not at all
2
3
4
5
Perfectly
99
Q

Location of the sublingual space

A

The sublingual space is located superomedial to the mylohyoid muscle

How well did you know this?
1
Not at all
2
3
4
5
Perfectly
100
Q

How are infections of the submandibular space produced?

A

Involvement of the submandibular space is produced principally by infections of the second and third mandibular molars because of the more superior position of the mylohyoid ridge on the mandible posteriorly, which places the root apices of the second and third molars beneath the mylohyoid muscle.

How well did you know this?
1
Not at all
2
3
4
5
Perfectly
101
Q

Extension of infections of the maxillary molars

A

Infections of the maxillary molars, when they extend through the buccal cortical plates above the attachments of the buccinator muscle, can present as infections of the buccal space.

How well did you know this?
1
Not at all
2
3
4
5
Perfectly
102
Q

Extension of infections from the anterior mandibular teeth

A

Infections from the anterior mandibular teeth (anterior to the second molar) usually drain above the mylohyoid muscle into the sublingual space.

How well did you know this?
1
Not at all
2
3
4
5
Perfectly
103
Q

The application of a locking reconstruction plate to a comminuted mandibular fracture is LEAST likely to cause which of the following?
(A) Decreased bone resorption
(B) More difficulty in contouring the plate
(C) Hardware failure
(D) Hardware-related infection
(E) Malocclusion

A

(E) Malocclusion

Use of a locking reconstruction bone plate has been shown to decrease postoperative malocclusion after a comminuted fracture of the mandible. A conventional (nonlocking) bone plate requires precise adaptation of the plate to the underlying bone. Without intimate contact, the bone is drawn toward the plate when the screws are tightened, altering the position of the osseous segments and the occlusal relationship. However, a locking bone plate does not require intimate contact of plate to bone because the bony segments are secured by screws that are locked to the plate. This makes it less likely for screw insertion to alter the reduction and, ultimately, the occlusion.

How well did you know this?
1
Not at all
2
3
4
5
Perfectly
104
Q

A conventional plate is also called a:

A

A conventional plate is a nonlocking plate

How well did you know this?
1
Not at all
2
3
4
5
Perfectly
105
Q

A conventional plate requires what positioning relative to the bone?

A

A conventional (nonlocking) bone plate requires precise adaptation of the plate to the underlying bone

How well did you know this?
1
Not at all
2
3
4
5
Perfectly
106
Q

What happens if a conventional plate is not precisely adapted to the underling bone?

A

Without intimate contact, for a conventional/nonlocking plate, the bone is drawn toward the plate when the screws are tightened, altering the position of the osseous segments and the occlusal relationship.

How well did you know this?
1
Not at all
2
3
4
5
Perfectly
107
Q

Positioning of a locking vs nonlocking plate vs the bone

A

Conventional/nonlocking plate requires precise adaptation of the plate to the underlying bone

A locking plate does not require intimate contact of the palte to the bone

How well did you know this?
1
Not at all
2
3
4
5
Perfectly
108
Q

Why doesn’t a nonlocking plate require precise positioning of the plate vs the bone?

A

A locking bone plate does not require intimate contact of plate to bone because the bony segments are secured by screws that are locked to the plate.

How well did you know this?
1
Not at all
2
3
4
5
Perfectly
109
Q

Implications of a nonlocking plate not requiring precise adaptation to the bone

A

This makes it less likely for screw insertion to alter the reduction and, ultimately, the occlusion.

110
Q

Use of a locking reconstruction bone plate has been shown to ________ postoperative malocclusion after a comminuted fracture of the mandible.

A

Use of a locking reconstruction bone plate has been shown to decrease postoperative malocclusion after a comminuted fracture of the mandible.

111
Q

Difficulty in plate contouring: Locking vs nonlocking plates

A

Difficulty in plate contouring is less likely to occur with locking plates because they require less precise bending than do conventional plates, which depend on intimate bony contact for stability.

112
Q

Likelihood of loosening screws: Locking versus nonlocking plates

A

Screws in locking plates are less likely to become loose than those in standard (nonlocking) reconstruction plate

113
Q

Rate of hardware-related infection: Locking versus nonlocking plates

A

The rate of hardware-related infection with locking plates is similar to the rate with standard reconstruction plates

114
Q

Likelihood of cortical compression: Locking versus nonlocking plates

A

Cortical compression occurs less frequently with locking plates than with standard reconstruction plates

115
Q

Likelihood of blood supply disruption: Locking versus nonlocking plates

A

Blood supply disruption occurs less frequently with locking plates than with standard reconstruction plates

116
Q

Likelihood of associated bone resorption: Locking versus nonlocking plates

A

Associated bone resorption occurs less frequently with locking plates than with standard reconstruction plates

117
Q
In planning open reduction and internal fixation in a patient with a low subcondylar neck fracture, which of the following extraoral incisions provides the safest and most versatile exposure to the fracture site?
(A) Postauricular
(B) Preauricular
(C) Retromandibular
(D) Submandibula
A

(C) Retromandibular

The retromandibular incision provides the safest and most versatile exposure for open reduction and internal fixation of submandibular fractures. When compared with the subcondylar and preauricular incisions, there is significantly less injury to the marginal mandibular, temporal, and zygomatic branches of the facial nerve.

The retromandibular incision allows access superiorly to the coronoid notch and inferiorly to the angle of the mandible. The addition of a transfacial trocar to this approach facilitates access to higher level subcondylar fracturesas well. The preauricular, postauricular, and submandibular incisions provide a more limited view of low subcondylar fractures

118
Q

What provides the safest/most versatile exposure for open reduction and internal fixation of submandibular fractures?

A

The retromandibular incision provides the safest and most versatile exposure for open reduction and internal fixation of submandibular fractures.

119
Q

When compared with the subcondylar and preauricular incisions, morbidity of retromandibular incisions:

A

When compared with the subcondylar and preauricular incisions, there is significantly less injury to the marginal mandibular, temporal, and zygomatic branches of the facial nerve.

120
Q

The retromandibular incision allows access to:

A

The retromandibular incision allows access superiorly to the coronoid notch and inferiorly to the angle of the mandible.

121
Q

With the addition of a transfacial trochar, the retromandibular incision allows access to:

A

The retromandibular incision allows access superiorly to the coronoid notch and inferiorly to the angle of the mandible. The addition of a transfacial trocar to this approach facilitates access to higher level subcondylar fracturesas well.

122
Q
A 5-year-old child has malocclusion and limited opening of the mouth after falling from playground equipment. A CT scan is shown (right subcondylar neck fracture). Which of the following is the most likely long-term sequela of this patient’s injury?
(A) Bimaxillary prognathism
(B) Mandibular hypoplasia
(C) Mandibular prognathism
(D) Maxillary hypoplasia
(E) Maxillary prognathism
A

(B) Mandibular hypoplasia

This child is at increased risk for mandibular hypoplasia. The condyle serves as a growth center for the mandible and contributes primarily to vertical growth. The condylar cartilage is a site of secondary passive growth dependent on forces acting on it, notably the medial and lateral pterygoid muscles. Pediatric condylar fractures generally remodel and do not often cause growth disturbance. However, the thin, localized functional matrix of the condyle may disallow normal mandibular growth after it has been injured and may result in unilateral or bilateral hypoplasia depending on the injury. Pediatric mandibular fractures are frequently treated conservatively via closed reduction and short periods of maxillomandibular fixation.

123
Q

The ______ is the growth center of the mandible and primarily contributes to _______ growth.

A

The condyle serves as a growth center for the mandible and contributes primarily to vertical growth.

124
Q

Potential sequelae of pediatric condylar fractures

A

Pediatric condylar fractures generally remodel and do not often cause growth disturbance. However, the thin, localized functional matrix of the condyle may disallow normal mandibular growth after it has been injured and may result in unilateral or bilateral hypoplasia depending on the injury.

125
Q

Two sites of growth of the mandible

A

The condyle serves as a growth center for the mandible and contributes primarily to vertical growth.

The condylar cartilage is a site of secondary passive growth dependent on forces acting on it, notably the medial and lateral pterygoid muscles

126
Q

What is growth at the condylar cartilage dependent upon?

A

The condylar cartilage is a site of secondary passive growth dependent on forces acting on it, notably the medial and lateral pterygoid muscles

127
Q

Frequently, pediatric mandible fractures are treated via:

A

Pediatric mandibular fractures are frequently treated conservatively via closed reduction and short periods of maxillomandibular fixation.

128
Q

A 12-year-old boy is brought to the emergency department after he fell while riding his bike and landed on the chin. Panorex radiographs show a minimally displaced fracture of the high right condylar neck and an open left parasymphyseal fracture. Which ofthe following is the most effective management?
(A) Observation with serial radiographs and restriction to soft diet
(B) Intermaxillary fixation with infraorbital and circummandibular wires for four weeks
(C) Intermaxillary fixation with arch bars for four weeks
(D) Open reduction with internal fixation of the parasymphyseal fracture and arch bars for two weeks
(E) Open reduction with internal fixation of both the parasymphyseal fracture and the fracture of the high condylar neck

A

(D) Open reduction with internal fixation of the parasymphyseal fracture and arch bars for two weeks

Observation alone is inadequate management of the parasymphyseal fracture. Open reduction and internal fixation (ORIF) of the parasymphyseal fracture will ensure an anatomic reduction of a stable skeletal unit. This, in turn, will decrease the risk of infection and nonunion. This child presents in the late phase of the mixed dentition and should have enough adult dentition in place to secure the arch bars

Immobilization for a short period (i.e., two weeks) is the appropriate management of the condylar neck fracture. This will help to allow the fractures to become stable enough to maintain the reduction once movement is instituted. Early movement helps to decrease the risk of ankylosis of the temporal mandibular joint.

129
Q

When are wires for a parasymphyseal fracture useful?

A

When a child does not have enough adult dentition to secure arch bars

130
Q

Appropriate management of a minimally displaced condylar neck fracture

A

Immobilization for a short period (i.e., two weeks) is the appropriate management of the condylar neck fracture. This will help to allow the fractures to become stable enough to maintain the reduction once movement is instituted.

131
Q

Risk of prolonged immobilization for a condylar neck fracture

A

Ankylosis!

Early movement helps to decrease the risk of ankylosis of the temporal mandibular joint.

132
Q
A 22-year-old man sustains a left subcondylar fracture of the mandible during a motor vehicle collision. On CT scan, the condyle is displaced medially and anteriorly. This displacement is most likely caused by tension from which of the following muscles?
(A) Medial pterygoid
(B) Lateral pterygoid
(C) Masseter
(D) Mylohyoid
(E) Temporalis
A

(B) Lateral pterygoid

The inferior belly of the lateral pterygoid originates from the lateral pterygoid plate and inserts onto the scaphoid fossa of the condyle and joint capsule. The superior belly of the lateral pterygoid muscle originates from the sphenoid and inserts on the temporomandibular joint. The effect of the lateral pterygoid muscle is to displace the condyle medially and anteriorly in fractures of the condylar neck. It also tends to displace the meniscus anteriorly. The muscles inserting directly on the mandible exert significant forces on fracture fragments. An understanding of their direction of pull and insertions is important in the proper reduction and fixation of mandibular fractures. All the muscles of mastication serve to elevateand protrude the mandible. The elevators include the masseter, medial pterygoid, and temporalis. The temporalis inserts onto the coronoid process and the superior aspect of the external oblique line. The masseter inserts onto the lateral aspect of the mandibular angle while the medial pterygoid inserts on the medial aspect of the mandibular angle. None of these muscles directly affect the condyle. The mylohyoid muscle inserts on the body of the mandible, displacing segmental body fractures medially.

133
Q

Effect of the lateral pterygoid in condylar neck fractures

A

The effect of the lateral pterygoid muscle is to displace the condyle medially and anteriorly in fractures of the condylar neck. It also tends to displace the meniscus anteriorly.

134
Q

Elevators of the mandible

A

The elevators include the masseter, medial pterygoid, and temporalis.

135
Q

A 22-year-old man sustains a transverse, noncomminuted fracture of the right mandibular angle when he is struck in the face during a fistfight. Which of the following interventions best adheres to Champy’s principle for management of this fracture?
(A) Dynamic compression plate with bicortical screws on the inferior edge of the mandible and a superior tension band
(B) Dynamic compression plate with bicortical screws and a mandibular arch bar
(C) Lag screw
(D) Miniplate with monocortical screws along the external oblique ridge
(E) Reconstruction plate with bicortical screws

A

(D) Miniplate with monocortical screws along the external oblique ridge

Champy’s principles for fracture management call for placement of miniplates along the lines of tension in the mandible at the site of the fracture. Because compression is not necessary, the miniplates can be anchored with monocortical screws. Based on the muscular forces pulling on the mandible, Champy determined that, anterior to the canine tooth, two miniplates are needed to control the rotational forces of the genial and digastric muscles; posterior to the canine tooth, just one miniplate is required.

In a 10-year review examining various methods for treating fractures of the mandibular angle, Ellis concluded that useof a single 2.0-mm noncompression miniplate was associated with fewer complications than a double-plate system (one using two compression or noncompression plates) or a reconstruction plate.

136
Q

Champy’s principles for fracture mangement call for:

A

Champy’s principles for fracture management call for placement of miniplates along the lines of tension in the mandible at the site of the fracture.

137
Q

Champy fixation: Type of screws and why

A

Because compression is not necessary, the miniplates can be anchored with monocortical screws.

138
Q

Champy: When are one vs two miniplates required?

A

Anterior to the canine tooth: two miniplates are needed to control the rotational forces of the genial and digastric muscles

Posterior to the canine tooth: just one miniplate is required.

139
Q

How did Champy determine when one vs two miniplates are required for mandible fractures?

A

Based on the muscular forces pulling on the mandible,

140
Q

Ellis’s determination of single versus double plate systems

A

In a 10-year review examining various methods for treating fractures of the mandibular angle, Ellis concluded that use of a single 2.0-mm noncompression miniplate was associated with fewer complications than a double-plate system (one using two compression or noncompression plates) or a reconstruction plate.

141
Q

A 26-year-old man comes to the office because he has pain in the mandible seven days after undergoing open reduction and internal fixation of a fracture of the mandible. Physical examination shows infection in the submandibular space. Which of the following teeth are the most likely source of this infection?
(A) Mandibular canines
(B) Mandibular central incisors
(C) Mandibular first and second premolars
(D) Mandibular second and third molars
(E) Maxillary second and third molars

A

(D) Mandibular second and third molars

The submandibular space is located inferolateral to the mylohyoid muscle and superior to the hyoid bone. The contents of the submandibular space include the submandibular gland, lymph nodes, the facial vein and artery, and the inferior loop of the hypoglossal (XII) nerve. Anteriorly, the submandibular space communicates with the submental space and posteriorly with the pharyngeal space. The sublingual space is located superomedial to the mylohyoid muscle. Involvement of the submandibular space is produced principally by infections of the second and third mandibular molars because of the more superior position of the mylohyoid ridge on the mandible posteriorly, which places the root apices of the second and third molars beneath the mylohyoid muscle. Infections of the maxillary molars, when they extend through the buccal cortical plates above the attachments of the buccinator muscle, can present as infections of the buccal space. Infections from the anterior mandibular teeth (anterior to the second molar) usually drain above the mylohyoid muscle into the sublingual space

142
Q

The submandibular space is located:

A

The submandibular space is located inferolateral to the mylohyoid muscle and superior to the hyoid bone.

143
Q

Contents of the submandibular space include:

A

The contents of the submandibular space include the submandibular gland, lymph nodes, the facial vein and artery, and the inferior loop of the hypoglossal (XII) nerve.

144
Q

The submandibular space communicates with:

A

Anteriorly, the submandibular space communicates with the submental space and posteriorly with the pharyngeal space.

145
Q

The sublingual space is located:

A

The sublingual space is located superomedial to the mylohyoid muscle.

146
Q

When do dental infections involve the submandibular space?

A

Involvement of the submandibular space is produced principally by infections of the second and third mandibular molars because of the more superior position of the mylohyoid ridge on the mandible posteriorly, which places the root apices of the second and third molars beneath the mylohyoid muscle.

147
Q

Infections of the maxillary molars can extend into:

A

Infections of the maxillary molars, when they extend through the buccal cortical plates above the attachments of the buccinator muscle, can present as infections of the buccal space.

148
Q

Infections of teeth anterior to the second molar drain into:

A

Infections from the anterior mandibular teeth (anterior to the second molar) usually drain above the mylohyoid muscle into the sublingual space

149
Q

The application of a locking reconstruction plate to a comminuted mandibular fracture is LEAST likely to cause which of the following?
(A) Decreased bone resorption
(B) More difficulty in contouring the plate
(C) Hardware failure
(D) Hardware-related infection
(E) Malocclusion

A

(E) Malocclusion

Use of a locking reconstruction bone plate has been shown to decrease postoperative malocclusion after a comminuted fracture of the mandible. A conventional (nonlocking) bone plate requires precise adaptation of the plate to the underlying bone. Without intimate contact, the bone is drawn toward the plate when thescrews are tightened, altering the position of the osseous segments and the occlusal relationship. However, a locking bone plate does not require intimate contact of plate to bone because the bony segments are secured by screws that are locked to the plate. This makes it less likely for screw insertion to alter the reduction and, ultimately, the occlusion

150
Q
A 67-year-old man has an ulcerated lesion of the anterior floor of the mouth with exposed, desiccated mandible one year after undergoing surgical resection and radiotherapy for squamous cell carcinoma of the anterior floor of the mouth. Initial recovery from the procedure was uncomplicated. Pathologic evaluation of a specimen obtained on excisional biopsy shows osteoradionecrosis. Radiation doses greater than 6500 cGy and which of the following are the most likely precipitating factors in this patient?
(A) Dental caries
(B) Dental implants
(C) Edentulous mandible
(D) Oral candidiasis
(E) Xerostomia
A

(A) Dental caries

Osteoradionecrosis (ORN) is relatively uncommon. However, the risk of ORN increases when the radiation dosage to the mandible exceeds 6500 cGy. Although up to 30% of cases of ORN reportedly arise spontaneously, most reports note dental caries and extraction sites as precipitating factors. Acute and chronic periodontal disease in mandibulotomy sites can also lead to ORN.Dental implants, an edentulous mandible, oral candidiasis, and xerostomia may all be seen in cases of mandibular reconstruction and radiation. However, they do not increase the risk of developing ORN.Traditional treatment of ORN includes surgical debridement and antibiotic therapy if infection is present. In advanced, extensive ORN, hyperbaric oxygen (HBO) therapy may be used as an adjunct. However, controversy exists about using HBO therapy instead of surgical resection and reconstruction of the mandible. The decision to use HBO therapy should be made on an individual basis and with the understanding that necrotic bone with sequestrum is unlikely to heal with HBO therapy

151
Q

At what does of RT does osteoradionecrosis of the jawmore often occur?

A

Osteoradionecrosis (ORN) is relatively uncommon. However, the risk of ORN increases when the radiation dosage to the mandible exceeds 6500 cGy.

152
Q

Etiologies of osteoradionecrosis of the jaw:

A
  • Most reports note dental caries and extraction sites as precipitating factors
  • 30% of cases of ORN reportedly arise spontaneously
  • Acute and chronic periodontal disease in mandibulotomy sites can also lead to ORN
153
Q

Factors that do not increase risk of osteoradionecrosis of the jaw

A

Dental implants, an edentulous mandible, oral candidiasis, and xerostomia may all be seen in cases of mandibular reconstruction and radiation. However, they do not increase the risk of developing ORN.

154
Q

Traditional treatment of ORN of the jaw

A

Traditional treatment of ORN includes surgical debridement and antibiotic therapy if infection is present.

155
Q

Hyperbaric oxygen therapy for osteoradionecrosis of the jaw

A

In advanced, extensive ORN, hyperbaric oxygen (HBO) therapy may be used as an adjunct. However, controversy exists about using HBO therapy instead of surgical resection and reconstruction of the mandible. The decision to use HBO therapy should be made on an individual basis and with the understanding that necrotic bone with sequestrum is unlikely to heal with HBO therapy

156
Q

Dental implants and risk of developing ORN of the jaw

A

Does not increase the risk of developing ORN.

157
Q

Edentulous mandible and risk of developing ORN of the jaw

A

Does not increase the risk of developing ORN.

158
Q

Oral candidiasis and risk of developing ORN of the jaw

A

Does not increase the risk of developing ORN.

159
Q

Xerostomia and risk of developing ORN of the jaw

A

Does not increase the risk of developing ORN.

160
Q

Xerostomia

A

Dry mouth

161
Q
Which of the following terms best describes the type of occlusion in which the upper central incisor lies anterior to the lower central incisor in the sagittal plane?
(A) Buccal crossbite
(B) Lingual crossbite
(C) Open bite
(D) Overbite
(E) Overjet
A

(E) Overjet

Overjet is a horizontal measurement that refers to the distance between the incisal aspect of the maxillary incisors and the incisal aspect of the mandibular incisors with the teeth in centric occlusion. When the upper central incisor lies anterior to the lower central incisor in the sagittal plane, this is known as overjet.In contrast, overbite is a vertical measurement referring to the distance between the maxillary incisor edge and the mandibular incisor edge with the teeth in centric occlusion. An overbite or deep bite is one in which the upper central incisor overrides the lower central significantly in the vertical dimension.Buccal and lingual crossbite refer to the positioning of the mandibular molars with respect to the maxillary molars in the transverse plane.Open bite occurs when the maxillary and mandibular teeth fail to contact. This can occur at any point in the dentition.

162
Q

What measurement does overjet refer to?

A

Overjet is a horizontal measurement that refers to the distance between the incisal aspect of the maxillary incisors and the incisal aspect of the mandibular incisors with the teeth in centric occlusion.

163
Q

Overjet in a patient:

A

When the upper central incisor lies anterior to the lower central incisor in the sagittal plane, this is known as overjet.

164
Q

What measurement does overbite refer to?

A

Overbite is a vertical measurement referring to the distance between the maxillary incisor edge and the mandibular incisor edge with the teeth in centric occlusion.

165
Q

Overbite in a patient:

A

An overbite or deep bite is one in which the upper central incisor overrides the lower central significantly in the vertical dimension.

166
Q

Buccal and lingual crossbite

A

Buccal and lingual crossbite refer to the positioning of the mandibular molars with respect to the maxillary molars in the transverse plane.

167
Q

Open bite

A

Open bite occurs when the maxillary and mandibular teeth fail to contact. This can occur at any point in the dentition.

168
Q

Re: Buccal and lingual crossbite, the _______ molars are being compared to the _______ molars in the transverse plane

A

positioning of the mandibular molars with respect to the maxillary molars

169
Q
Removal of a tooth in a fracture line of the mandible is indicated in a patient with which of the following conditions?
(A) Cavities in the tooth
(B) Fracture of the root of the tooth
(C) Loose tooth
(D) Multiple fractures of the mandible
(E) Periodontal disease
A

(B) Fracture of the root of the tooth

Indications for removal of teeth in mandibular fractures include fracture of the root of the tooth, severe loosening of the tooth in presence of chronic periodontal disease, extensive periodontal injury and broken alveolar walls, and displacement of teeth from their alveolar socket. Periodontal disease alone is not an indication for tooth removal. Multiple fractures of the mandible are also not an indication for tooth removal because the teeth usually are needed for intermaxillary fixation prior to open reduction and internal fixation of the fractures. History of caries would warrant a referral to a dentist to ascertain whether any intervention would be required but would not necessitate removal of that tooth at the time of fracture management. Loose tooth is seen in most cases of mandibular fracture but is addressed by proper alignment and reduction of all fractures

170
Q

Indications for removal of teeth in mandibular fractures:

A
  • Facture of the root of the tooth
  • Severe loosening of the tooth in presence of chronic periodontal disease
  • Extensive periodontal injury and broken alveolar walls
  • Displacement of teeth from their alveolar socket.
171
Q

Mandible fractures: Periodontal disease as an indication for tooth removal

A

Periodontal disease alone is not an indication for tooth removal.

172
Q

Mandibular fractures: Multiple fractures as an indication for tooth removal

A

Multiple fractures of the mandible are also not an indication for tooth removal because the teeth usually are needed for intermaxillary fixation prior to open reduction and internal fixation of the fractures.

173
Q

Mandibular fractures: Loose tooth as an indication for tooth removal

A

Loose tooth is seen in most cases of mandibular fracture but is addressed by proper alignment and reduction of all fractures.

Severe loosening of the tooth in presence of chronic periodontal disease IS an indication.

174
Q

In distraction osteogenesis of the mandible using an external distractor, successful formation of the bone is most dependent on which of the following?
(A) Consolidation period of two weeks
(B) Distraction rate of 1 mm per day
(C) Lag period of three days before initiation of distraction
(D) Stable fixation of the bone
(E) Supraperiosteal dissection of the bone

A

(D) Stable fixation of the bone

In distraction osteogenesis of the mandible using an external distractor, the most important element in successful formation of the bone is adequate stabilization of the bone edges. Unstable fixation of the bone allows excessive motion, which can result in a fibrousunion.

A consolidation period of two weeks is inadequate. A period of at least four to six weeks usually is needed before the distraction devices can be safely removed. Although a rate of 1 mm per day commonly is used in mandibular distraction, rates of2 mm or more per day have been shown to be successful in mandibular distraction, particularly when an external device is used. Most surgeons use a lag period before beginning distraction. However, a lag period has never been demonstrated to be necessary in craniofacial distraction. Supraperiosteal dissection of the bone is important in distraction of the extremities but has not been demonstrated to be necessary in distraction of the mandible.

175
Q

In distraction osteogenesis of the mandible using an external distractor, the most important element in successful formation of the bone is:

A

In distraction osteogenesis of the mandible using an external distractor, the most important element in successful formation of the bone is adequate stabilization of the bone edges. Unstable fixation of the bone allows excessive motion, which can result in a fibrousunion.

176
Q

Consolidation period for mandibular distraction: Duration; this is important when determining when to:

A

A period of at least four to six weeks usually is needed before the distraction devices can be safely removed.

177
Q

Accepted rates of mandibular distraction

A

Although a rate of 1 mm per day commonly is used in mandibular distraction, rates of 2 mm or more per day have been shown to be successful in mandibular distraction, particularly when an external device is used.

178
Q

Supraperiosteal dissection of the bone and mandibular distraction

A

Supraperiosteal dissection of the bone is important in distraction of the extremities but has not been demonstrated to be necessary in distraction of the mandible.

179
Q

Lag period for mandibular distraction

A

Most surgeons use a lag period before beginning distraction. However, a lag period has never been demonstrated to be necessary in craniofacial distraction.

180
Q

In distraction osteogenesis of the mandible using an external distractor, unstable fixation of the bone can result in a fibrousunion.

A

Unstable fixation of the bone allows excessive motion, which can result in a fibrousunion.

181
Q

A 25-year-old man who sustained a fracture of the maxillary alveolus involving the right central and lateral incisors is scheduled to undergo operative reduction and application of an arch bar in the emergency department for stabilization of the fracture. Adequate local anesthesia in this patient involves blockade of which of the following nerves?
(A) Greater palatine and anterior superior alveolar
(B) Greater palatine and buccal
(C) Nasopalatine and anterior superior alveolar
(D) Nasopalatine and buccal
(E) Infraorbital and middle superior alveolar

A

(C) Nasopalatine and anterior superior alveolar

This 25-year-old man is to undergo operative reduction of a fracture of the maxillary alveolus involving the right central and lateral incisors, followed by application of an arch bar. To obtain a sensory blockade, the surgeon must anesthetize the nasopalatine nerve to block the palate (lingual surface) and the anterior superior alveolar nerve to block the teeth and alveolar mucosa (buccal surface). The nasopalatine nerve originatesfrom the infraorbital nerve and passes through the incisive foramen to reach the anterior hard palate. It innervates the premaxilla at this point, then extends posteriorly to innervate the maxillary cuspids. The anterior superior alveolar nerve branches from the infraorbital nerve after it exits the infraorbital foramen and provides innervation to the maxillary incisors and canine teeth.

182
Q

To obtain a sensory blockade for maxillary alveolar fracture / arch bar application

A

To obtain a sensory blockade, the surgeon must anesthetize the nasopalatine nerve to block the palate (lingual surface) and the anterior superior alveolar nerve to block the teeth and alveolar mucosa (buccal surface).

183
Q

The nasopalatine nerve: Anatomy and innervation to:

A

The nasopalatine nerve originates from the infraorbital nerve and passes through the incisive foramen to reach the anterior hard palate. It innervates the premaxilla at this point, then extends posteriorly to innervate the maxillary cuspids

184
Q

The anterior superior alveolar nerve: Anatomy and innervation to:

A

The anterior superior alveolar nerve branches from the infraorbital nerve after it exits the infraorbital foramen and provides innervation to the maxillary incisors and canine teeth.

185
Q

The greater palatine nerve: Anatomy and innervation to:

A

The greater palatine nerve emerges from the greater palatine foramen and provides sensation to the posterior portion of the hard palate.

186
Q

The buccal nerve provides sensation to:

A

The buccal nerve provides sensation to the buccal mucosa and lower gingiva.

187
Q

The middle and posterior superior alveolar nerves are derived from the ____________ when:

A

The middle and posterior superior alveolar nerves are derived from the infraorbital nerve after it exits the pterygopalatine fossa.

188
Q

The middle superior alveolar nerve innervates:

A

The middle superior alveolar nerve innervates the bicuspids

189
Q

The posterior superior alveolar nerve innervates:

A

The posterior superior alveolar nerve innervates the first, second, and third molars within the maxilla

190
Q

The anterior superior alveolar nerve branches from:

A

The infraorbital nerve, after it exits the infraorbital foramen

191
Q

In a 27-year-old man who has sustained bilateral parasymphyseal fractures, which of the following muscles exerts a distractive force on the anterior fracture segment?
(A) Geniohyoid
(B) Lateral pterygoid
(C) Masseter
(D) Medial pterygoid
(E) Posterior belly of the digastric muscle

A

(A) Geniohyoid

Both anterior and posterior muscles exert forces on the mandible. The anterior muscles consist of the geniohyoid, genioglossus, mylohyoid, and digastric muscles. The muscles from this group exert primary distractive forces on the anterior fracture segment of a parasymphyseal fracture, displacing the fracture segment downward, posteriorly, and medially. The geniohyoid muscle originates from the mental spine of the inner anterior mandible and inserts on the hyoid bone. It acts to depress and retract the mandible.The masseter, temporalis, andmedial and lateral pterygoids comprise the posterior muscles. These muscles do not exert any force on the anterior segment of a parasymphyseal fracture.The digastric muscle has an anterior and a posterior belly. The posterior belly originates on the medial aspect of the mastoid and courses forward and inferiorly as a tendon, passing through a fascial sling on the hyoid to transition into the anterior belly, which inserts into the digastric fossa of the mandible. The posterior belly of the digastric muscleprimarily functions to elevate the hyoid and exhibits only a secondary effect on the anterior mandible. In contrast, the anterior belly of the digastric muscle exerts force on the anterior fracture segment.

192
Q

Anterior muscles of the mandible

A

The anterior muscles consist of the geniohyoid, genioglossus, mylohyoid, and digastric muscles.

193
Q

Parasymphyseal fracture: What forces to the anterior group apply on the anterior segment of the mandible?

A

Distractive forces: displacing the fracture segment downward, posteriorly, and medially.

194
Q

Geniohyoid anatomy

A

The geniohyoid muscle originates from the mental spine of the inner anterior mandible and inserts on the hyoid bone.

195
Q

Actions of the geniohyoid on the mandible

A

It acts to depress and retract the mandible.

196
Q

Posterior digastric muscle anatomy

A

The posterior belly originates on the medial aspect of the mastoid and courses forward and inferiorly as a tendon, passing through a fascial sling on the hyoid to transition into the anterior belly, which inserts into the digastric fossa of the mandible.

197
Q

Actions of the posterior belly of the digastric muscle

A

The posterior belly of the digastric muscle primarily functions to elevate the hyoid and exhibits only a secondary effect on the anterior mandible.

198
Q

Parasymphyseal mandibular fracture: Anterior vs posterior muscle bellies of the digastric

A

The posterior belly of the digastric muscle primarily functions to elevate the hyoid and exhibits only a secondary effect on the anterior mandible.

In contrast, the anterior belly of the digastric muscle exerts force on the anterior fracture segment.

199
Q
Simultaneous ipsilateral contractions of which of the following muscles produce the side-to-side grinding movements of the mandible?
(A) Masseter and lateral pterygoid
(B) Masseter and medial pterygoid
(C) Masseter and temporalis
(D) Medial and lateral pterygoid
(E) Medial pterygoid and temporalis
A

(D) Medial and lateral pterygoid

Simultaneous contractions of the medial and lateral pterygoid muscles largely produce the side-to-side grinding and chewing movements of the mandible. Both pairs of medial and lateral pterygoid muscles have two heads. The heads of the medial pterygoids originate from the medial surface of the lateral pterygoid plate and the tuberosity of the maxilla and insert into the medial surface of the mandibular angle and ramus. These muscles act to elevate the mandible. The heads of the lateral pterygoids originate from the lateral surface of the lateral pterygoid plate and the infratemporal surface of the greater wing of the sphenoid and insert into the neck of the mandibular condyle and articular disk. These muscles act to protrude the mandible forward and open the mouth. When the ipsilateral medial and lateral pterygoid muscles work together, rotation occurs around the vertical axis of the contralateral condyle. Grinding and chewing movements occur when both sides alternate this action in rhythmic fashion. Simultaneous action of all four pterygoid muscles results in protrusion of the mandible.

200
Q

What produces the side-to-side grinding and chewing movements of the mandible?

A

Simultaneous contractions of the medial and lateral pterygoid muscles largely produce the side-to-side grinding and chewing movements of the mandible.

201
Q

Origin of the medial pterygoids

A

The heads of the medial pterygoids originate from the medial surface of the lateral pterygoid plate and the tuberosity of the maxilla

202
Q

Insertion of the medial pterygoids

A

Inserts into the medial surface of the mandibular angle and ramus.

203
Q

Action of the medial pterygoids

A

These muscles act to elevate the mandible.

204
Q

Origin of the lateral pterygoids

A

The heads of the lateral pterygoids originate from the lateral surface of the lateral pterygoid plate and the infratemporal surface of the greater wing of the sphenoid

205
Q

Insertion of the lateral pterygoids

A

Inserts into the neck of the mandibular condyle and articular disk.

206
Q

Action of the lateral pterygoids

A

These muscles act to protrude the mandible forward and open the mouth.

207
Q

When the ipsilateral medial and lateral pterygoid muscles work together, what happens?

A

When the ipsilateral medial and lateral pterygoid muscles work together, rotation occurs around the vertical axis of the contralateral condyle.

208
Q

Pterygoid muscles: Grinding and chewing movements occur when

A

Grinding and chewing movements occur when both sides alternate this action in rhythmic fashion.

209
Q

Simultaneous action of all four pterygoid muscles results in:

A

Simultaneous action of all four pterygoid muscles results in protrusion of the mandible.

210
Q

The masseter muscle arises from:

A

The masseter muscle arises from the lower border and medial surface of the zygomatic arch

211
Q

The masseter attaches to:

A

The masseter attaches to the lateral aspect of the mandibular ramus.

212
Q

The masseter functions to:

A

It functions primarily to elevate the mandible to occlude the teeth.

213
Q

The temporalis muscle originates in:

A

The temporalis muscle originates in the temporal fossa of the cranium

214
Q

The temporalis muscle attaches to:

A

The temporalis muscle attaches to the coronoid process of the mandible.

215
Q

Although its primary function is elevation of the mandible, the temporalis muscle can also:

A

Although its primary function is elevation of the mandible, the temporalis muscle can also retract the mandible because of the action of the posterior muscle fibers.

216
Q
Which of the following nerves supplies sensory innervation to the buccal mucosa?
(A) Trigeminal (V) nerve
(B) Facial (VII) nerve
(C) Glossopharyngeal (IX) nerve
(D) Vagus (X) nerve
(E) Lingual nerve
A

(A) Trigeminal (V) nerve

The buccal branch of the trigeminal (V) nerve provides sensation to the buccal mucosa. It is important for the surgeon to know the anatomy of this nerve branch to plan and perform neurotized free flap reconstruction and reinnervation of the intraoral cavity.
The buccal branch of the facial (VII) nerve innervates the muscles surrounding the buccal mucosa. The glossopharyngeal(IX) and vagus (X) nerves do not provide sensory innervation to the intraoral mucosa.
The lingual nerve provides sensation to a portion of the tongue

217
Q

What provides sensory innervation to the buccal mucosa?

A

The buccal branch of the trigeminal (V) nerve provides sensation to the buccal mucosa.

218
Q
A 29-year-old man is involved in a fistfight and sustains bilateral fractures of the mandible in the region of the canine teeth. On examination, the fractures are vertically and horizontally unstable. The central segment of the mandible is most likely to be displaced in which of the following directions?
(A) Downward and anterior
(B) Downward and posterior
(C) Rotationally
(D) Upward and anterior
(E) Upward and posterior
A

(B) Downward and posterior

This 29-year-old man has sustained bilateral fractures of the mandible that are located in the region of the canine teeth and are unstable both vertically and horizontally. Therefore, the central segment is most likely displaced downward and posteriorly due to the action of the anterior bellies of the digastric muscles and the geniohyoid and genioglossus muscles. In addition, the power of the mylohyoid muscle pulls the posterior fracture segments medially.

Rotational displacement of the central segment may occur if the fracture is stable either vertically or horizontally because the action of the muscles would then be opposed partially. Rotational displacementis characterized by hinging of the central segment on the upper or lower border of the mandible.Upward and anterior displacement occurs only if the central fragment becomes detached from surrounding muscles, which is unlikely

219
Q

Bilateral fractures of the mandible: When do rotational displacements of the central segments occur?

A

Rotational displacement of the central segment may occur if the fracture is stable either vertically or horizontally because the action of the muscles would then be opposed partially. Rotational displacementis characterized by hinging of the central segment on the upper or lower border of the mandible.

220
Q
Which of the following muscles of mastication pulls the mandible upward, medially, and forward?
(A) Anterior digastric
(B) Masseter
(C) Medial pterygoid
(D) Mylohyoid
(E) Temporalis
A

(C) Medial pterygoid

The medial pterygoid muscle exerts upward, medial, and forward traction on the mandible. This muscle arises inferiorly, laterally, and posteriorly from the medial surface of the pterygoid plate and inserts on the medial ramus and mandibular angle. Contraction of the medial pterygoid muscle pulls the mandible medially, elevating the lower jaw. A patient who sustains a fracture medial to the mandibular angle has displacement of the mandibular ramus medially and cephalad because of the forces of this muscle.

221
Q

A patient who sustains a fracture medial to the mandibular angle has displacement of the mandibular ramus ________ because of the forces of the _________

A

A patient who sustains a fracture medial to the mandibular angle has displacement of the mandibular ramus medially and cephalad because of the forces of the medial pterygoid.

222
Q

In patients with zygomatic fractures, displacement of the zygoma is exacerbated by:

A

In patients with zygomatic fractures, displacement of the zygoma is exacerbated by the chronic active traction of the masseter muscle.

223
Q

Role that the temporalis muscle may play in displacing bone fragments in patients with subcondylar fractures?
This would result in:

A

The temporalis muscle elevates and retracts the mandible. In patients with subcondylar fractures, this muscle may elevate the mandibular ramus, resulting in loss of posterior facial height.

224
Q
According to the Angle classification, which of the following permanent teeth are used to determine the class of dental occlusion?
(A) Central incisors
(B) Lateral incisors
(C) First premolars
(D) Second premolars
(E) First molars
A

(E) First molars

According to the Angle classification, the permanent first molars are used to determine the class of dental occlusion. Angle class I, or normal, occlusion is defined as the mesiobuccal cusp of the maxillary first molar lying in the buccal groove of the mandibular first molar. In Angle class II malocclusion, the mesiobuccal cusp of the maxillary first molar lies mesial (anterior) to the buccal groove of the mandibular first molar. Angle class III malocclusion is defined as the mesiobuccal cusp of the maxillary first molar lying distal (posterior) to the buccal groove of the mandibular first molar and being located instead in the buccal groove of the lower second molar.

225
Q

Angle class I malocclusion

A

Angle class I, or normal, occlusion is defined as the mesiobuccal cusp of the maxillary first molar lying in the buccal groove of the mandibular first molar.

226
Q

Angle class II occlusion

A

In Angle class II malocclusion, the mesiobuccal cusp of the maxillary first molar lies mesial (anterior) to the buccal groove of the mandibular first molar.

227
Q

Angle class III malocclusion

A

Angle class III malocclusion is defined as the mesiobuccal cusp of the maxillary first molar lying distal (posterior) to the buccal groove of the mandibular first molar and being located instead in the buccal groove of the lower second molar.

228
Q

The ____________ are used to determine overbite and overjet.

A

The central incisors are used to determine overbite and overjet.

229
Q

The central incisors are used to determine: (anatomical relationship)

A

The central incisors are used to determine overbite and overjet.

230
Q
A 25-year-old woman is undergoing evaluation of microgenia. Cephalometric analysis shows marked bony deficiencies in both the anteroposterior and vertical dimensions. Occlusion is normal. Which of the following is the most appropriate management?
(A) Chin implantation
(B) Intraoral vertical osteotomy
(C) Sagittal split osteotomy
(D) Osseous genioplasty
(E) Distraction osteogenesis
A

(D) Osseous genioplasty

In this patient who has microgenia with anteroposterior and vertical deficiencies of the chin, the most appropriate management is osseous genioplasty, which will increase the projection of the mental symphysis within the sagittal plane while compensating for the vertical deficiency. Microgenia is one of several terms used to describe abnormalities of the chin. In this condition, the chin is small and bone is deficient in all three planes.

Both intraoral vertical osteotomy and sagittal split osteotomy of the mandibular ramus will alter dental occlusion andare not indicated for correction of vertical or horizontal deficiencies, especially in a patient who has normal occlusion.

Distraction osteogenesis is indicated for treatment of skeletal deficiencies (ie, Pierre Robin sequence) but is an unnecessary, excessive option for correction of this deformity.

231
Q

Microgenia

A

In this condition, the chin is small and bone is deficient in all three planes; can be associated with normal occlusion or mandibular prognathism.

232
Q

Macrogenia

A

Macrogenia describes a large chin; can be associated with normal occlusion or mandibular prognathism.

233
Q

Retrogenia

A

In patients with retrogenia, the chin is positioned posteriorly; it is not necessarily small.

234
Q

Pure vs secondary retrogenia

A

Occlusion is normal in pure retrogenia. Patients with mandibular retrognathia have secondary retrogenia.

235
Q

Chin implantation is recommended to:

It is not indicated for:

A

Chin implantation is recommended to increase the anteroposterior projection of the chin
Chin implantation does not correct vertical deficiency or excess.

236
Q

Management of vertical microgenia

A

Moderate-to-severe vertical microgenia frequently includes anteroposterior deficiency of the chin.

This abnormality can be corrected by performing osseous genioplasty, which will increase the projection of the mental symphysis within the sagittal plane while compensating for the vertical deficiency.

237
Q
A 31-year-old man sustains a fracture of the condylar neck of the mandible. On physical examination, the condylar fragment is displaced medially. This finding is most likely caused by the action of which of the following muscles? 
A) Buccinator
(B) Lateral pterygoid
(C) Masseter
(D) Medial pterygoid
(E) Temporalis
A

(B) Lateral pterygoid

Fragments of fractures of the condylar neck are displaced by the action of the lateral pterygoid muscle.

The masseter, medial pterygoid, and temporalis muscles all insert below the fracture line within the condyle. Therefore, these muscles do not exert any pull on the condylar segment.

238
Q

Fragments of fractures of the condylar neck are displaced medially by:

A

Fragments of fractures of the condylar neck are displaced medially by the action of the lateral pterygoid muscle.

239
Q

The masseter, medial pterygoid, and temporalis muscles: actions on the condylar section, in a condylar neck fracture

A

The masseter, medial pterygoid, and temporalis muscles all insert below the fracture line within the condyle. Therefore, these muscles do not exert any pull on the condylar segment.

240
Q

A patient has an infection at the surgical site one week after undergoing open reduction and internal fixation of a fracture of the mandibular body using an inferior border reconstruction plate and a tension band. Occlusion is normal. The infection site is surgically drained; intraoperative exploration shows that the plates and screws are stable with no evidence of loosening. Which of the following is the most appropriate management of the hardware?
(A) Maintenance of current stabilization without removal of the hardware
(B) Removal of all plates and immediate application of intermaxillary fixation
(C) Removal of all plates and immediate application of two miniplates
(D) Removal of all plates and immediate placement of an external fixator
(E) Removal of all plates and placement of new plates when the infection has subsided

A

(A) Maintenance of current stabilization without removal of the hardware

Infections following open reduction and internal fixation of mandibular fractures typically result from failure of fixation devices, for example, loosening of the screws. In addition to operative drainage of the infection and antibiotic therapy, appropriate management in the majority of these situations includes removal and replacement of the hardware with intermaxillary or external fixation for stabilization of the fracture. In this patient, the plates continue to provide stable fixation of the fracture. Because of this, the current stabilization should be maintained without removal of the hardware, and the patient should undergo operative drainage of the infection and administration of antibiotics.

241
Q

Infections following open reduction and internal fixation of mandibular fractures typically result from:

A

Infections following open reduction and internal fixation of mandibular fractures typically result from failure of fixation devices, for example, loosening of the screws.

242
Q

Appropriate management in the majority infection following ORIF of mandibular fractures

A

In addition to operative drainage of the infection and antibiotic therapy, appropriate management in the majority of these situations includes removal and replacement of the hardware with intermaxillary or external fixation for stabilization of the fracture.

243
Q

Indications for maintaining hardware after infection, s/p ORIF of a mandibular fracture

A

While most situations will require hardware removal, if stable fixation of the fracture is demonstrated, the current stabilization should be maintained without removal of the hardware, and the patient should undergo operative drainage of the infection and administration of antibiotics.

244
Q
A 36-year-old woman has pain in the right side of the mandible after falling and striking her face on a sidewalk. Physical examination shows numbness of the right lower lip. The most likely cause of these findings is a fracture of which of the following regions of the mandible?
(A) Ascending ramus
(B) Body
(C) Condyle
(D) Coronoid process
(E) Symphysis
A

(B) Body

These findings are most consistent with a fracture of the body of the mandible. The mandibular angle and body are fractured most commonly. Fractures of the mandibular body are most likely to cause impingement or transection of the inferior alveolar nerve, resulting in numbness of the right lower lip. This nerve enters the mandible at a point proximal to the mandibular angle, travels through the angle and body to the mental foramen opposite the first bicuspid tooth, then emerges as the mental nerve, supplying sensation to the soft tissues of the lip and chin. Physical examination and radiographs are most likely to confirm the location of the fracture.Fractures of the ascending ramus, condyle, coronoid process, and symphysis are less common than fractures of the mandibular angle and body and would not result in numbness of the lower lip

245
Q

Fractures of the ____________ are most likely to cause impingement or transection of the inferior alveolar nerve, resulting in:

A

Fractures of the mandibular body are most likely to cause impingement or transection of the inferior alveolar nerve, resulting in numbness of the lower lip (unilateral)

246
Q

Which fractures are most likely to cause numbness of the lower lip?

A

Fractures of the mandibular body are most likely to cause impingement or transection of the inferior alveolar nerve, resulting in numbness of the lower lip (unilateral to fracture)

247
Q

Interior alveolar nerve: anatomy

A

Interior alveolar nerve: This nerve enters the mandible at a point proximal to the mandibular angle, travels through the angle and body to the mental foramen opposite the first bicuspid tooth, then emerges as the mental nerve, supplying sensation to the soft tissues of the lip and chin.

248
Q

Body and angle

A

Which two fractures are most common in for adult mandibular fractures?

249
Q

Mental nerve: What is its origin?

A

The inferior alveolar nerve ravels through the angle and body to the mental foramen opposite the first bicuspid tooth, then emerges as the mental nerve

250
Q

The mental nerve supplies:

A

Sensation to the soft tissues of the lip and chin.

251
Q

Where is the mental foramen?

A

Opposite the first bicuspid tooth

252
Q

A 10-year-old boy has a laceration of the chin and pain in the jaw and ear after falling while ice skating. On examination, the maximal incisal opening is 10 mm, and the chin point is deviated to the left. There is an upward cant of the mandibular occlusion on the left with a right-sided lateral open bite. These findings are most consistent with which of the following?
(A) Bilateral condylar fractures
(B) Bilateral temporomandibular joint dislocation
(C) Left-sided condylar fracture
(D) Left-sided mandibular body fracture
(E) Right-sided condylar fracture

A

(C) Left-sided condylar fracture

Although left-sided mandibular body fractures can be associated with limited mouth opening, a contralateral open bite and an ipsilateral upward occlusal cant are not typical of this type of fracture.As mentioned above, a right-sided condylarfracture would manifest as a left-sided lateral open bite with chin deviation and an upward occlusal cant on the right

The findings in this child are most consistent with a left-sided condylar fracture. It is necessary to exclude a diagnosis of condylar fracture in any child who sustains trauma to the chin. Indications for a diagnosis of condylar fracture include malocclusion, pain with range of motion of the temporomandibular joint, and preauricular pain. Lacerations of the external auditory canal may also be associated. Patients with unilateral condylar fractures exhibit loss of posterior ramus height unilaterally, resulting in premature contact of the maxillary and mandibular molars posteriorly and a contralateral lateral open bite. The mandibular occlusal plane will demonstrate an ipsilateral upward cant. The maximal incisal opening will be decreased, and the chin point and mandibular midline will be deviated ipsilaterally due to the unopposed action of the lateral pterygoid muscle on the contralateral side. Because this child has chin deviation and an upward cant on the left with a right-sided lateral open bite, a left-sided condylar fracture can be diagnosed.

253
Q

Indications for diagnosis of condylar fracture, in general

A
  • Malocclusion
  • Pain with range of motion of the temporomandibular joint
  • Preauricular pain

Lacerations of the external auditory canal may also be associated.

254
Q

Clinical presentation of unilateral condylar fracture

A
  • Loss of posterior ramus height unilaterally, resulting in premature contact of the maxillary and mandibular molars posteriorly and a contralateral lateral open bite.
  • The mandibular occlusal plane will demonstrate an ipsilateral upward cant
  • The maximal incisal opening will be decreased
  • The chin point and mandibular midline will be deviated ipsilaterally due to the unopposed action of the lateral pterygoid muscle on the contralateral side.
255
Q

Clinical presentation of pediatric bilateral condylar fractures

A

A child with bilateral condylar fractures will have an anterior open bite resulting from premature contact of the mandibular and maxillary molars posteriorly. Ear pain and lacerations of the external auditory canal may also be present bilaterally.

256
Q

Bilateral temporomandibular joint dislocation typically results in:

A

Bilateral temporomandibular joint dislocation typically results in an open bite and severe limitation of jaw excursion, also known as “lock-jaw.”

257
Q

A 24-year-old woman sustains facial injuries in a motorvehicle collision. On examination, there is tenderness in the preauricular region bilaterally, posterior facial height is decreased, and there is malocclusion with an anterior open bite. Panoramic radiographs show low subcondylar fractures of the mandible bilaterally. The mandibular condyles are seated within the glenoid fossa, and the proximal segment overrides the distal segment laterally. Which of the following is the most appropriate management?
(A) Observation
(B) Intermaxillary fixation for two weeks followed by physical therapy
(C) Intermaxillary fixation for eight weeks followed by physical therapy
(D) Bilateral external fixation
(E) Open reduction and internal fixation

A

(E) Open reduction and internal fixation

Although a short course of intermaxillary fixation (two to three weeks) followed by graduated opening of the mandible has traditionally been implemented in the management of subcondylar fractures, it does not address fracture malalignment or its potential complications. Prolonged intermaxillary fixation (six weeks or more) is associated with an increased risk for temporomandibular joint stiffness and a subsequent decrease in interincisal opening

Because stable anatomic reduction of the fracture segments is crucial for management of this patient’s injuries, open reduction and internal fixation should be performed via a preauricular approach. Accurate reduction of a subcondylar fracture is rarely achieved with closed reduction alone. In addition, the absence of internal fixation will lead to fracture instability secondary to the forces of the masseter, temporalis, and medial and lateral pterygoid muscles, ultimately resulting in decreased posterior facial height and abnormal condylar mechanics caused by displacement of the condylar head. The patient will be at greater risk for malocclusion and development of degenerative osteoarthritis. Therefore, accurate open reduction with rigid internal fixation is advocated to avoid any potential complications. With this approach, normal posterior facial height will be restored, and the risk for abnormal joint mechanics will be minimized. Endoscopically-assisted fracture reduction, with rigid fixation, is a new technique that shows promise because it combines the advantages of the open approach (ie, anatomic reduction and early motion) while minimizing external scarring and the risk for facial nerve injury.

258
Q

Approach for reduction of displaced subcondylar fractures

A

Preauricular

259
Q

Why is a displaced subcondylar fracture rarely able to be successfully treated with a closed approach?

A
  • Accurate reduction of a subcondylar fracture is rarely achieved with closed reduction alone.
  • The absence of internal fixation will lead to fracture instability secondary to the forces of the masseter, temporalis, and medial and lateral pterygoid muscles, ultimately resulting in decreased posterior facial height and abnormal condylar mechanics caused by displacement of the condylar head.

–> Greater risk for malocclusion and development of degenerative osteoarthritis.

260
Q

Management of displaced subcondylar fracture

A

ORIF:

  • Restore normal posterior facial height
  • Minimize risk of abnormal joint mechanics
261
Q
In pediatric patients, abnormalities in mandibular growth are most closely associated with fractures involving which of the following regions of the mandible?
(A) Angle
(B) Body
(C) Condyle
(D) Ramus
(E) Symphysis
A

(C) Condyle

In children, abnormalities in mandibular growth are most closely associated with fractures involving the mandibular condyle. More than one-third of all facial fractures in children involve the mandible. The pediatric condyle, which is the primary growth center of the mandible, is immature, highly vascular, and covered with a thin sheath of periosteum. Any compressive or traumatic forces may cause the condyle to burst, rather than fracture, resulting in fragmentation of bone, hemarthrosis, and increased osteogenic potential

262
Q
A 6-year-old boy is brought to the emergency department following facial trauma from falling on his bicycle handlebars. Which of the following mandible fracture locations is most commonly associated with anterior open bite?
A) Angle
B) Body
C) Coronoid
D) Subcondylar/condylar
E) Symphysis/parasymphysis
A

D) Subcondylar/condylar

Anterior open bite, also known as apertognathia, is vertical separation of the maxillary and mandibular anterior teeth. It is caused by premature contact of the posterior molars, most commonly following bilateral subcondylar mandible fracture. When present, a unilateral subcondylar/condylar fracture causes an open bite on the side opposite the fracture.

263
Q

A 62-year-old woman who underwent chemotherapy/radiation protocol for oropharyngeal cancer 10 years ago has onset of severe pain after a dental extraction. Subsequent CT scan shows a pathologic fracture of the mandibular angle. Which of the following is the most appropriate management?
A) Hyperbaric oxygen therapy
B) Long-term intravenous antibiotic therapy
C) Oncology consultation
D) Open reduction and internal fixation of the mandible
E) Resection and coverage with a fibular free flap

A

E) Resection and coverage with a fibular free flap

Over the past few decades, the use of chemotherapy/radiation as the primary curative treatment for oropharyngeal cancer has increased. In part, this has to do with cure rates and tissue preservation, but it is also due to the rise in human papillomavirus–positive oropharyngeal cancer. Not surprisingly, there has been a marked increase in osteoradionecrosis of the mandible, in particular. The most common cause of pathologic fracture after radiation therapy in the mandible is tooth extraction, usually the third molar, and a subsequent angle injury, as in this case. With a large, multi-decade experience in the use of osseous free flaps, especially the fibular flap, these cases are now routinely managed by resection of the affected bone and immediate reconstruction.

Hyperbaric oxygen, as a single modality for osteoradionecrosis, is at best controversial and would not cure a pathologic fracture.

Intravenous antibiotics can treat osteomyelitis, but in a case of osteoradionecrosis and a fracture, a short course of adjuvant antibiotics (along with appropriate surgery) would be sufficient, at best.

Open reduction and internal fixation of this fracture would not suffice either, because necrotic bone will not heal, even if put into juxtaposition.

Although it is important to assume that any pathology in cases like these are cancerous until proven otherwise, nevertheless, this scenario as described is very common and the constellation of events plus the imaging indicates that oncology’s role in this case would be limited at best.

264
Q
A 25-year-old man comes to the office because of jaw pain after sustaining a punch to the face 2 days ago. Maxillofacial CT scan shows a displaced comminuted fracture of the left mandibular angle. Open reduction and internal fixation is planned. Which of the following methods of fixation is considered load-bearing osteosynthesis?
A) Champy plate (oblique ridge)
B) Compression plates
C) Lag screws
D) Locking reconstruction plate
E) Simple screws
A

D) Locking reconstruction plate

Load-bearing osteosynthesis of the mandible may be accomplished with a reconstruction plate and locking screws.

There are two basic types of mandibular fracture fixation: load-bearing osteosynthesis and load-sharing osteosynthesis.

In load-bearing osteosynthesis, the plate (or external fixator) assumes all the forces of mandibular function at the fracture site. Common clinical indications include comminuted fractures, fractures with segmental defects, and those in the atrophic edentulous mandible.

In load-sharing osteosynthesis, stability at the fracture site is created by the frictional resistance between the end of the bone and the hardware used for fixation. This requires adequate bony buttressing across the fracture line. Lag screws, Champy plate (at the oblique ridge), compression plates, and simple screws are examples of load-sharing osteosynthesis, which may have different levels of force distribution between the hardware and the bone.

265
Q
In a trauma patient with an isolated mandible fracture, which of the following is the likelihood that this patient will have a concomitant cervical spine injury?
A) 1%
B) 5%
C) 15%
D) 25%
E) 50%
A

B) 5%

In a recent review of the National Trauma Data Bank of over 1.3 million trauma patients, an analysis of isolated facial fractures and neurologic injury was undertaken. An isolated mandible fracture had a 5.1% relationship with a cervical spine injury. These data showed a higher than previously published rate of cervical spine and head injures associated with isolated facial fractures.

266
Q

Eventual management of a plate in plated parasymphyseal fracture

A

Removal 2-3 months following surgery, to prevent growth restriction and embedding of the plate

267
Q

A 25-year-old man comes to the office for treatment of malocclusion 6 days after he was involved in an altercation. Physical examination shows right-sided facial swelling. X-ray study shows an unfavorable fracture through the angle of the right mandible. No other associated injuries are noted. Which of the following is the most appropriate treatment?
A) Closed reduction and maxillomandibular fixation
B) Open reduction and maxillomandibular fixation
C) Open reduction and rigid fixation
D) Open reduction, wire fixation, and maxillomandibular fixation
E) Observation only

A

C) Open reduction and rigid fixation

Mandible fractures are a frequent injury because of the mandible’s prominence and relative lack of support. Numerous investigators have reported studies on populations on all continents; fractures of the mandible have been reported to account for 36 to 70% of all maxillofacial fractures. All reports apparently show a higher frequency in males aged 21 to 30 years. In patients with mandible fractures, 53% of patients had unilateral fractures, 37% of the patients had two fractures, and 9% had three or more fractures. Fractures sustained in altercations – low-energy fractures – tend to have single, simple patterns.

The indications for closed versus open reduction have changed dramatically over the last century. The ability to treat fractures with open reduction and rigid internal fixation (ORIF) has dramatically revolutionized the approach to mandibular fractures.

Traditionally, closed reduction (CR) and ORIF with wire osteosynthesis have required an average of 6 weeks of immobilization by maxillomandibular fixation for satisfactory healing. Difficulties associated with this extended period of immobilization include airway problems, poor nutrition, weight loss, poor hygiene, phonation difficulties, insomnia, social inconvenience, patient discomfort, work loss, and difficulty recovering normal range of jaw function.

In contrast, rigid and semirigid fixation of mandible fractures allow early mobilization and restoration of jaw function, airway control, improved nutritional status, improved speech, better oral hygiene, patient comfort, and an earlier return to the workplace.

268
Q

A 55-year-old woman is referred to the office by her dentist because of a 6-week history of exposed intraoral bone. She takes zoledronic acid for osteoporosis. Physical examination shows a 1-cm ulceration of gingiva with exposed necrotic bone adjacent to the right premolar. No infection or fistulization is noted. In addition to meticulous oral hygiene, which of the following is the most appropriate management?
A) Administration of prophylactic oral antibiotics
B) Curettage and bone grafting
C) Dental extraction
D) Segmental resection
E) Observation only

A

E) Observation only

The most appropriate management is observation only. The clinical vignette illustrates a case of Stage I bisphosphonate-related osteonecrosis of the jaw (BRONJ). These patients are typically asymptomatic, with the exception of exposed and/or necrotic bone. Antibiotics are not recommended unless there is infection (Stage II or III). Stage II BRONJ features exposed and/or necrotic bone with pain and local infection.

Curettage and bone grafting is not appropriate.

Dental extractions are risky and may trigger exacerbation. In cases of serious tooth decay, endodontics (root canal) and crown amputation is preferred.

Segmental resection is reserved for Stage III BRONJ. In general, the need for surgery is guided by the severity of the stage. Stage III BRONJ is characterized by exposed and/or necrotic bone with pain, infection, and the presence of another complication, such as osteolysis extending from the superior to the inferior border of the mandible, pathologic fracture, or extraoral fistula.

269
Q

A 33-year-old woman is brought to the emergency department after sustaining injuries in a motor vehicle collision. The patient notes pain on opening her mouth. Physical examination shows bilateral facial swelling and loss of posterior facial height. An anterior open bite is also noted. Which of the following additional findings on physical examination is most likely to suggest a bilateral subcondylar fracture in this patient?
A) Bilateral facial numbness
B) Bilateral mastoid ecchymosis
C) Blood in the external auditory canal
D) Clear fluid in the external auditory canal
E) Preauricular pain

A

E) Preauricular pain

Bilateral subcondylar fractures result in premature occlusion of the posterior teeth along with an anterior open bite, loss of posterior facial height, and bilateral facial swelling with pain on mouth opening. These findings occur because the subcondylar fracture interrupts the integrity of the vertical buttress. The lateral pterygoid muscles displace the condylar necks medially and anteriorly, allowing the unopposed vertical action of the temporalis and masseter muscles to shorten the posterior facial height. Swelling bilaterally would be expected in this fracture, and opening would displace the fracture line, causing pain.

Bilateral facial numbness suggests a fracture of the ramus or body as the inferior alveolar nerve traverses these areas. Blood in the external auditory canal suggests a fracture more proximal than subcondylar. Bilateral mastoid ecchymosis and/or clear fluid in the external auditory canal suggest a skull base fracture.

270
Q
A 2-month-old male infant is brought to the office because of mid face hypoplasia, craniosynostosis, and bilateral hand and foot anomalies. A photograph of the left foot is shown (broad great toe and synostosis of other toes). This patient most likely has which of the following syndromes?
A) Apert
B) Crouzon
C) Goldenhar
D) Nager
E) Treacher Collins
A

A) Apert

The patient described has Apert syndrome. This autosomal dominant syndrome is characterized by bicoronal craniosynostosis that leads to turribrachycephaly, mid face hypoplasia, and complex hand and feet syndactyly. Patients with Crouzon syndrome, an autosomal dominant disorder, typically have craniosynostosis involving the coronal, sagittal, and lambdoid sutures, as well as turribrachycephaly. Other findings include mid face hypoplasia, exorbitism, and proptosis. The extremities are normal.

Goldenhar syndrome, or oculoauriculovertebral dysplasia, involves asymmetry of the hard and soft tissues of the face. This condition is most commonly unilateral but may be seen bilaterally in some patients. Manifestations of this syndrome include hypoplasia involving the mandible and underlying soft tissues of the face, epibulbar dermoids, and varied degrees of microtia on the affected side. Most patients have associated vertebral abnormalities. Nager syndrome, or acrofacial dysostosis, is an autosomal recessive disorder characterized by craniofacial and upper extremity abnormalities. Patients with Nager syndrome have hypoplasia of the orbits, zygoma, maxilla, mandible, and soft palate. Auricular defects may also be present. Hypoplasia or agenesis occurs in the radius, thumbs, and metacarpals. Some patients may have radioulnar synostosis and elbow joint deformities. Patients with Treacher Collins syndrome, or mandibular dysostosis, have hypoplasia of the zygoma, maxilla, and mandible, downward slanting of the palpebral fissures, colobomas of the lower eyelids, absence of eyelashes, and auricular defects.